LMR Georgette’s PMHNP Certification Exam Latest 2023-2024 Questions And Correct Answers(Verified Answers)

Which patient is at highest risk for SI

A. 30y/o married AA female with previous SI attempt *1 risk factor

B. 35 y/o single Asian male with previous SI attempt *3 risk factors

C. 38 y/o single AA male who is a manager of a bank *2 risk factors

D. 68 y/o single white male with depression *5 risk factors (age, male, white, depression)

D. 68 y/o single white male with depression *5 risk factors (age, male, white, depression)

Count the risk factors

When interview teenagers (16 y/o) that arrive with their parents what should you do?
interview them separately from parents.
-This helps Build therapeutic rapport with teens by telling them the info is confidential. Parents may be upset but remember you are advocating for the child.

Which Ethnic group has the highest rate of suicide?
Native Americans

Example A patient is being treated for schizophrenia with olanzapine. Which of the following is the most common side effect of olanzapine?
A. Increased waist circumference
B. EPS (not as common in atypical antipsychotics d/t 5HT2A)-receptor antagonism
C. Increased Lipids
D. Metabolic Syndrome
D. Metabolic Syndrome (UMBRELLA ANSWER)

Which antipsychotics have the least weight gain?
Latuda, Abilify, (also least sedating), Geodon-if patient has metabolic syndrome consider switching to one of the medications above. Or if the patient is overly sedated try switching to ABILIFY

Which mood stabilizer have the least weight gain?
Lamictal
-But remember all mood stabilizers cause some weight gain

When presented with a question about typical vs atypical antipsychotic the answer is usually to start of a
atypical

A client presents with complains of changes in appetite, feeling fatigued, problems with sleep-rest cycle, and changes in libido. What is the neuroanatomical area of the brain that is responsible for the normal regulation of these functions?

A. Thalamus

B. Hypothalamus

C. Limbic System

D. Hippocampus

Hypothalamus
A, B, & D are all part of the limbic system so you can rule that out

When a patient is hesitant to participate in treatment you should encourage?
Bring a support person like a husband

Thyroid-Stimulating hormone normal level
0.5-5.0 Mu/L

When T4 and T3 are high and TSH is low what is the diagnosis
HYPERTHYROIDISM, TSH secretion decreases: TSH LOW à key symptoms HEAT INTOLERANCE

Key symptoms of Heat Intolerance
Hyperthyroidism

When T4 and T3 are Low and TSH is high what is the diagnosis
(HYPOTHYROIDISM) TSH secretion increased: TSH HIGH à COLD INTERANCE

Key symptoms of Cold Intolerance
Hypothyroidism

Hyperthyroid can mimic
Mania

Hypothyroid can mimic
Depression

A patient on depakote complains of RUQ pain and has reddish/brown urine
Hepatoxicity
-Check LFTs

Signs of Depakote toxicity
Disorientation, confusion, lethargy

You suspect depakote toxicity what do you do?
Check
-LFT
-Ammonia
-Depakote Level

What herbal supplement can cause hepatoxicity?
Kava Kava

When taking Kava Kava in combinations with other medications you should caution about
Risk of Hepatoxicity and Sedation

TCAs carry a risk of
Hepatotoxicity

Signs of Stevens-Johnson Syndrome
-fever, mouth pain, swelling, burning eyes, blisters, skin pain

two psychotropics known to cause steven johnson syndrome
lamictal and tegretol

What nationality is most suseptible of getting steven johnson?
Asians

When treating asians with tegretal screen for?
HLAB-1502 Allele

What two medications cause agranulocytosis?
Clozaril & Tegretal

Agranulocytosis when to discontinue medication
Less than 1000

When monitoring for agranulocytosis in patients look for s/s of what?
Infection
-Fever, sore throat, fatigue, chills

Before starting any mood stabilizer in a female of childbearing age be sure to check?
HCG

Which two medications may decrease the risk of suicide?
clozaril and lithium

Medications that increase lithium level
NSAID-ibuprofen, INDOCIN
THIAZIDES-hydrochlorithiazide ACE INHIBITORS-lisinopril

Ace inhibitors are treatment of choice for?
Heart Failure

Certain medications are known to increase lithium level, but HOW?
by reducing renal clearance

When educating a patient about lithium teach them about
Hyponatremia
Dehydration-hot days, exercise

Normal Lithium Level
0.6-1.2

Lithium Toxicity
1.5 or above
Discontinue and re-order lithium level

Lithium level of 1.4
Monitor for toxicity

Labs before starting lithium
TSH, BUN, CREATININE, HCG, U/A to check for presence of protein in the urine (4+ protein is concerning for renal impairment)à4+ protein in urine=MONITOR FOR TOXICITY

4+ protein in the urine of a patient on lithium
4+ protein is concerning for renal impairment

4+ protein in urine=MONITOR FOR TOXICITY

Lithium side effects
hypothyroid, leukocytosis, maculopapular rash, t-wave inversion, Coarse Hand Tremor, GI upset (nausea, vomiting, anorexia)

-Some of these are also signs of toxicity

Signs of lithium toxicity
confusion, ataxia, GI upset, palpitation, tremor

NMS
muscle rigidity, mutism (because of muscle rigidity), increased CPK (caused by muscle contraction and muscle destruction), increase WBC, increased WBC, myoglobinuria (also from muscle destruction)

Cherry colored urine in a patient that exercises a lot
test for myoglobinuria may be a sign of rhabdo

Serotonin Syndrome
With any drug that increases 5-HT (e.g., MAO inhibitors, SNRIs, TCAs) hyperthermia, confusion, myoclonus, cardiovascular instability, flushing, diarrhea, seizures.
-Treatment: cyproheptadine (5-HT2 receptor antagonist).

Treatment for NMS
Stop Offending Medication

-Dantrolene (muscle relaxer)

-Bromocriptine (Dopamine D2 agonist).

*In question focus on what they are asking for….dopamine agonist vs muscle relaxer

Treatment for Serotonin Syndrome
Stop Med (1 or more SSRI, SSNRI, TCA, MOAI)

-Cyproheptadine

Triptans
Used for MIGRAINES

-These meds increase serotonin

example SUMATRIPTAN

patient taking Prozac and started on sumatriptan

-call PCP to ask them to switch the migraine med if patient already on SUMATRIPTAN do not start antidepressant without talking to PCP

How long do you wait when switching between an SSRI to an MAOI?
2 weeks

How long do you wait when switching between Prozac and MAOI?
5-6 weeks wash out period

What is the first line treatment for depression and why?
SSRI-First line treatment for depression due to less risk of injury from OVERDOSE

If a cancer patient has depression what should you consider?
Treating with a medication with minimal drug/drug side effects like Lexapro

Patient with depression worries about sexual dysfunction what would be the medication of choice?
Wellbutrin

Primary symptoms of depression include fatigue and low energy what med would you chose?
Wellbutrin

Wellbutrin is contraindicated in patients with
Seizures and anorexia

Which medications are best for neuropathic pain?
SNRI
Gabapentin
TCA

Secondary to the black box warning providers caring for patients on antidepressants should assess for?
Suicidality, frequency, and severity at EVERY appointment

Which meds have the worse serotonin discontinuation syndrome
Those with short half lives
such as zoloft

Symptoms of serotonin withdrawal syndrome
Fever, achiness, soreness, lethargy, fatigue, impaired memory, decreased concentration, GI UPSET

Shits and Shivers

Ages of onset for schizophrenia in males vs females
-MALES 18-25 years

-FEMALE 25-35 years

Schizophrenia increases the risk for
SUICIDE

HIGH RISK OF SI in SCHIZOPHRENIA

Just having schizophrenia increases your risk of suicide.

MUST ASK ABOUT SI, EVERYTIME (frequency, severity of thoughts)

What increases the causes or increases the risk or schizophrenia
excessive pruning of synapses

-inadequate synapse formation,

-intrauterine insults such as maternal exposure to toxins, viral agents, maternal substance use, maternal illness, maternal malnutrition, fetal oxygen deprivation,

-first order relative (mom/dad)

MRI or PET scan what is seen in schizophrenia
EVERYTHING DECREASES EXCEPT VENTRICLES

-You will see VENTRICULAR ENLARGEMENT

Stimulants can potentiate the release of what neurotransmitter?
Dopamine which can worsen symptoms of schizophrenia

Assertive Community Treatment (ACT)
a form of rehabilitation post hospitalization, in home treatment

What level of prevention is ACT?
Tertiary

What adjunctive treatment is important in schizophrenia
-social skills training
-Exercise

Exercise for mental health patients can promote
Cognition
Quality of Life
Long-term health

ACT is ideal for patients with a history of
Treatment non-compliance

-Think about making the treatment convenient for them–>bringing it to their home

What diagnosis has the highest risk of Homicidality
Antisocial

In the MMSE how do you test for abstraction?
proverb interpretation (everyone that lives in glass houses shouldn’t throw stones) Are they able to think abstractly

Thought Process-Tangential
means that their response has nothing to do with the question

Circumstantial
means that their response goes in circles instead of getting to the point of the question

Mental Status-Thought Content includes
SI/HI/AH/VH

Another name for MMSE
Folstein Scale

How to assess concentration on MMSE
Serial 7s or perform an activity backwards i.e list the days of the week backwards

Assess ability to learn new material
repeat 3 words after me

Assess ability to recall
repeat 3 words after 5 minutes

Assess fund of knowledge
Who is the president

What is a quick and easy way to assess for neurological issues
Clock drawing test

If patient is unable to draw a clock this indicates
Problem with the right hemisphere, cerebrum, or parietal lobe

mesolimbic pathway
Hyperactivity of dopamine in the this pathway mediates positive psychotic symptoms

-Antagonism of D2 receptors in this pathway treats positive psychotic symptoms

mesocortical pathway
-Decreased dopamine in the this projection to the dorsolateral prefrontal cortex is postulated to be responsible for negative and depressive symptoms of schizophrenia

Nigrostriatal Pathway
-This pathway mediates motor movements

-Dopamine blockade in this pathway can lead to increase acetylcholine levels

-Blockade of dopamine (D2) receptors in this pathway can lead to EPS, i.e dystonia, parkinsonian symptoms and akathisia

Low Dopamine in the nigrostriatal pathway increases which neurotransmitter
-Dopamine has a reciprocal relationship with acetylcholine (Ach) (LOW DOPAMINE INCREASE Ach)

Long-standing D2 blockade in the nigrostriatal pathway can lead to

tardrive dyskinesia

Tuberoinfundibular pathway
-Blockade of D2 receptors in this pathway can lead to increase prolactin levels leading to hyperprolactinemia which clinically manifests as amenorrhea, galactorrhea, and sexual dysfunction, gynecomastia

-DECREASE DOPAMINE INCREASED PROLACTIN

Long-term hyperprolactinemia can be associated with what condition

osteoporosis

Normal Prolactin Level in Men
level less than 20ng/ml

Normal Prolactin Level in Women
less than 25ng/ml

Which medication is the highest offender for increasing prolactin
Risperdal

Acute Dystonia + Treatment
neck stiffness, muscle spasm of upper body especially neck/face/tongue

-Treatment is IM COGENTIN + continue PO COGENTIN for several days

Akathisia + Treatment
may mimic anxiety, restlessness, can’t sit still, rocking, pacing

-First line Treatment is BETA-BLOCKERS like PROPANOLOL (Inderal)

-Second line treatment is COGENTIN

-Third line treatment is benzos

Beta-Blockers such as Inderal are contraindicated with what type of asthma medication
-DO NOT GIVE WITH BROCHODIALATOR such as ALBUTERAL this combination can cause bronchospasm

akinesia/bradykinesia + treatment
A. difficulty initiating movement; slowness of movement
-Treatment Cogentin

PSEUDOPARKINSON or PARKINSONIAN + Treatment
caused by dopamine blockade, results in muscle rigidity, mask like facial expression, may look blunted, pill rolling tremors in fingers, shuffling gait, motor slowing

-Treatment COGENTIN

tardive dyskinesia + Treatment
abnormal facial movements, grinding teeth, lip smacking, protruding tongue

-Treatment DECREASE DOSE OF MED, DISCONTINUE MED, Switch to CLOZARIL, Switch to different med, VINPAT

Does Cogentin Treat TD
COGENTIN MAKES TD WORSE

Typical onset of TD
OCCURS 1-2 years TYPICALLY, but can be ACUTE ONSET ALSO

What non-psych med can cause TD?
REGLAN (Metoclopramide) can CAUSE Tardive Dyskinesia must educate patient that this med or the combination of this PLUS antipsychotic can increase risk of TD*** encourage them to discontinue reglan if TD develops

InDucers CYP450

DECREASE

Carbamazepine

Rifampin

Alcoholics (chronic)

Phenytoin

Grisiofulvin

Phenobarb

Sulphonylureas

Crap GPS Induces me to Madness!

InhIbitors of CYP450

INCREASE

Ciprofloxacin
Ritonavir
Amiodarone
Cimetidine
Ketoconazole

Acute Etoh
Macrolides
INH
Grapefruit Juice
Omeprazole

Crack Amigos

Erythromycin and Clarithromycin can cause
Increased tegretol levels

Patient started on Clozaril or Zyprexa and two months later starts smoking

as a provider you know that the smoking can decrease the medication effectiveness

-Increase medication dose

Patient has been a chronic smoker and has been stable on Zyrexa but tells you that he recently quit smoking cold turkey
as a provider you know that you must now decrease the dose of the antipyshcotic

Medications that cause mania
Steroids, Disulfiram (Antabuse), Isoniazid (INH), Antidepressants in persons with bipolar

-If a patient must take steroids, the provider should increase the mood stabilizer

Medications that cause depression
steroids, beta blockers, interferon, Accutane (isotrentinoin), some retroviral drugs, antineoplastic drugs, benzodiazepines, progesterone

-may need to increase antidepressant

Accutane (isotretinoin)
Can cause depression and birth defects

Flonase
As a provider you know that flonase is a STEROID so it may exacerbate mood symptoms

Increase mood stabilizer to maintain stability, steroids can also trigger depression

Flonase can trigger mood instability but it can also cause an increase in
Psychosis

patient is taking flonase while on antipsychotic but you find that the antipsychotic is ineffective it is likely because the flonase is exacerbating psychosis

-increase the dose of antipsychotic

Neurotransmitters involved in Addiction
Dopamine and GABA

Symptoms of Stimulant Abuse

  1. agitation/aggression
  2. impaired judgment
  3. euphoria
  4. elevated BP
  5. tachycardia
  6. dilated pupils
  7. hallucinations
  8. TREMORS
  9. IMSOMNIA

If an anorexic patient complains of pain or bloating after eating this may indicate
delayed gastric emptying

Medications that delay gastric emptying
Omeprazole, ranitidine, famotidine

Proton Pump Inhibitors (omeprazole & Protonix)
Decrease absorption of antipsychotics & SSRI

-MUST WAIT TWO HOURS BEFORE TAKING ANTIPSYCHOTIC OR SSRI

When initiating an SSRI on an elderly patient you should advise about
increased anxiety

Paradoxical effect
when meds cause the opposite effect than expected

Apoptosis
programmed cell death/neuronal loss

At age 45 and above the patient displays mania for first time what should be ruled out

MEDICAL CONDITION

Patient with bipolar disorder presents with depressed mood & emotional lability

Give Depakote

Hallmark sx of Borderline Personality
Recurrent self harm

Treatment for Borderline Personality
DBT

Creator of DBT
Marsha Linehan

What activity is helpful in making a diagnosis of borderline personality
Journaling or diary keeping

Conversion Disorder
STRESS leads to neurological symptoms such as seizures, paresthesia, blindness, mutism

Adjustment Disorder
adjusting to a situation resulting in depression or anxiety or both or mixed disturbance of emotions and conduct (this type is more common in children: insomnia, peer conflict, verbal altercations, truancy, crying)

-Symptoms occur within 3 months of the stressor

If question states recently moved, recent death….THINK ADJUSTMENT

factitious disorder
when patients introduce foreign substances into their body or contaminate their food

-Faking illness but NO MOTIVE BEHIND IT

Malingering
Faking illness for financial gain

Reactive Attachment
common in children in foster care, abuse from parents

-Withdrawn and shows no emotion towards caregiver

ODD
They deliberately annoy others, no aggression, defiance of authority

-Family Therapy is mainstay

-Child management /Parent management skills is the focus in therapy

-Positive reinforcement

-Boundary Setting

Conduct Disorder
violence, criminal, fire setting, killing animals, gang activity, +AGGRESSION, NO REMORSE

-May need meds and therapy

-Goal of therapy is to target MOOD & AGGRESSSION (mood stabilizers, antipsychotics, alpha agonists/alpha 2 adrenergic receptor blockers such as guanfacine and clonidine)

-Monitor BP with guanfacine and clonidine

Acute Stress Disorder
similar to PTSD but the timeline differs

-heightened arousal, nightmares, flashbacks

-LESS THAN ONE MONTH

PTSD
-OVER ONE MONTH

-3 HALLMARK SXS: intrusive re-experiencing of trauma, increased arousal, avoidance

-May also have NIGHTMARESà GIVE PRAZOSIN

-Non-pharm tx of PTSD- EMDR, CBT

Panic attack vs Panic disorder (treatment)
Panic attack = BZ
Panic disorder = SSRI

Panic Attack is ACUTE
Panic Disorder is CHRONIC

Feels like impending doom

Tourette’s Syndrome
Criteria for diagnosis

-TWO moto tics and ONE vocal tics

-LASTS more than ONE YEAR

-By age 18

CHILDREN MAY NORMALLY HAVE TICS so if they have one tic only THIS IS NORMAL

Child presents with one tic and the parent is worried
CHILDREN MAY NORMALLY HAVE TICS so if they have one tic only THIS IS NORMAL

Neurotransmitters involved in Tourettes
DNS: Dopamine, Norepinephrine, Serotonin

Treatment for tourettes
Treatment: Haldol, Pimozide, Abilify, Guanfacine, clonidine

What type of medication can cause tics or exacerbate them
Stimulants

Neurotransmitters involved in mood disorders
DNS: Dopamine, Norepinephrine, Serotonin + GABA

Neurotransmitters involved in ADHD
DNS: Dopamine, Norepinephrine, Serotonin

part of brain implicated in ADHD
prefrontal cortex
basal ganglia
reticular activating system

ADHD inattentive type is caused in what part of the brain
Prefrontal Cortex which is known to regulate ATTENTION and EXECUTIVE FUNCTION

dorsolateral prefrontal cortex
Attention
Executive Function
Cognition
Processing
Working Memory
Problem Solving

Deficit in the _ can lead to ADHD inattentive type
Prefrontal Cortex

Teacher reports that the stimulant only works for first few hours of class

medication has worn off too fast. Order multiple dosing throughout the day

When does the aftercare plan start
on admission

If parents become anxious while you are educating about a new diagnosis what should you do
-Provide patient and parents information immediately don’t wait till discharge

-Parents may become anxious after a diagnosis of mental illness such as ADHD, stop teaching offer support because they will not absorb the education. Provide supportive therapy

Neurotransmitters involved in OCD
serotonin, dopamine, glutamate & GABA

A tic may also be a _
Compulsion

Facts about OCD
Obsession/Compulsion

-A tic may be a compulsion

-If first order relative has OCD the child’s risk of developing OCD is increased

-Streptococcal infections increase risk of OCD

-Treatment SSRI-prozac, Zoloft, if adult you may also use TCA such as clomipramine

If question asks if the patient has Tourette’s vs OCD listen for mention of streptococcal treatment this will trigger you to think OCD

DMDD
6-17 years ONLY

-Irritability for no reason, sad, depressed mood, tantrums, crying, moody, always mad

If patient presents with irritability or labile mood and you need help further delineating symptoms
Administer MOOD QUESTIONAIRE
7/13 Bipolar Diagnosis Likely

Sleep Disorders are often _
So what should you assess if a parent reports that a child is having nightmares
GENETIC

ask if someone in the family has a similar issue with sleep…look for family patterns of sleep problems

GAD
Worry, apprehension, fear must LAST ATLEAST 6 MONTHS

Delirium
-ACUTE (within hours to days) onset of disturbance of LOC, COGNITION,

inattention

-Urinary Tract Infections are common cause for DELIRIUM always check UA

-Treatment is antipsychotics like HALDOL

Dementia
-Chronic and slow onset (months to years to develop)

-Mental decline in cognition, irritability, personality changes

-When asked questions they may try to answer or MAKE UP ANSWERS (confabulate)

Low levels of what labs may mimic dementia
Vit B12 and Folic Acid

Cortical Dementia
Language and memory (aphasia and amnesia)

Subcortical Dementia
Motor abnormalities/Mood issues like apathy, depression, irritability

HIV Dementia is a type of subcortical dementia

Early signs of HIV dementia
subcortical form of dementia

COGNITIVE, MOTOR, BEHEAVIOR for example a patient with lack of coordination, unsteady gait

Treatment for HIV dementia
Antivirals

Pseudo Dementia
Depression causes the memory issues, common in older adults

-Also assess onset of symptoms, pseudo dementia is more acute onset

-When asked questions they often say “I DON’T KNOW”

Instruments to use to differentiate between dementia and pseudo dementia
-Use instrument to further screen out cognitive issues such as SLUMS, MOCHA, MMSE

-Older individuals with depression may present with irritability and agitation

If question is asking you to differentiate between depression and dementia look at the amount of time that the symptoms have been present

hallmark of lewy body dementia
visual hallucinations

Frontotemporal lobe Dementia
PICKs Disease

-Hallmark is personality changes, language difficulties, poor impulse control, and behavioral changes

-May see slurred speech or difficulty getting words out

What lobe is associated with ability to understand what others are saying (comprehending speech)
Temporal Lobe

Neurotransmitters involved in Autism
GABA, Glutamate, Serotonin

Autism
a disorder that appears in childhood and is marked by deficient communication, social interaction, Poor eye contact, May not respond when you call their name, Stereotypical movement

When play they often like to line up their toys, stack them in tidy rows

Broken Mirror Theory of Autism
Explains that the child’s presentation is caused by the mirror neuron i.e dysfunction in the mirror neuron

Risk Factors for Autism
Male gender, genetic loading, intellectual disability, parents ages, preterm

Screening tools for Autism
ADOS-G (autism diagnostic observation schedule-genetic)
ASQ (ages and stages questionnaire)
M-CHAT (modified-checklist for autism-toddler)

Where is Norepinephrine produced?
locus coeruleus and medullary reticular formation

Where is serotonin produced?
raphe nuclei

Where is dopamine produced?
substantia nigra, ventral tegmental area, nucleaus accumbens

Where is acetylcholine synthesized?
Basal nucleus of Meynert

Hippocampus
a neural center located in the limbic system; helps process memory and manage stress

Limbic System
The limbic system is the part of the brain involved in our behavioral and emotional responses, especially when it comes to behaviors we need for survival: feeding, reproduction and caring for our young, and fight or flight responses.
-Hippocampus
-Amygdala
-Hypothalamus
-Thalamus

Amygdala function
Responsible for the response and memory of emotions, especially fear

Thalamus function
relay station for sensory impulses, pain

hypothalamus function
homeostasis, temperature, thirst, appetite, sex drive, sleep cycle, emotions

  • believed to serve a regulatory role in aggression

anterior cingulate cortex
brain region that regulates cognitive function, decision making, empathy, impulse control, and emotions

Cerebellum
Balance and coordination

signs of lead toxicity
developmental delay, learning diff., irritability, loss of appetite, weight loss, sluggishness, fatigue, abdominal pain, vomiting, constipation, hearing loss, seizures, eating non-food items PICA

Hint home built before 1970’s

TEST FOR LEAD

When caring for an infant that is about to die?
GIVE THE BABY TO THE PARENTS and allow them to grieve

Risk factors for osteoporosis
Age
smoking
caffeine
lack of exercise
diet lacking calcium and vit D

Provide Education

If discharging a patient that is not following up with outpatient care, organize ways to help the patient get to the appointment REMEMBER STAY INVOLVED IN THE CARE

Assume you are doing group therapy and there is a patient that is not comfortable sharing but you are trying to promote interpersonal learning. What should you do?

Provide adjunctive individual session that will help facilitate group participation

Cognitive Therapy
-Aaron Beck

Replacing irrational or distortive thoughts with positive thoughts

Behavioral Therapy
-Arnold Lazarus
focuses on changing behavior by identifying problem behaviors, replacing them with appropriate behaviors
-Exposure
-Relaxation
-Skills training
-Role Playing

Humanistic Therapy
-Carl Rogers
person-centered therapy
-Self-actualization
-Self-Directive Growth
-Everyone has the potential to actualize and find meaning in life

Existential Therapy
Victor Frankl -an insight therapy that focuses on the elemental problems of existence, such as death, meaning, choice, and responsibility, emphasized making courageous life choices.
-Emphasizes accepting freedom and making responsible choices
-Focus on the present

Why am I here, What is my purpose

Interpersonal Therapy
Gerald Kierman & Myrna Weissman

–Used for people who have trouble interacting with others, relationship distress

-Marital conflict

-12-16 weeks (3-4 months)

EMDR Phases
Desensitization Phase: visualize the trauma, verbalize negative thoughts but remain attentive to physical sensations

Installation Phase: Installs and increases strength of the positive thoughts that the patient has declared as a replacement

Body Scan: Visualize the trauma along with the positive thought and then scan ones body mentally to identify any tension within

Group therapy: Installation of hope
participants develop hope for creating a different life; they gain hope from others

Group therapy: Universality
people have similar problems, thoughts, and feelings and they are NOT ALONE

Group Therapy: Altruism
sharing of oneself with another and helping another

Group Therapy: Imitative Behavior
Patients can increase their skills by imitating the bx of others

Group Therapy: Interpersonal learning
interacting with others increases adaptive interpersonal relationships

Group Therapy: Group Cohesiveness
Patients develop an attraction to the group and other members as well as a sense of belonging

Group Therapy: Catharsis
Patients openly express their feelings which were previously suppressed

Group Therapy: Existential Factors
Groups enable participants to deal with the mean of their own existance

Group Therapy: Corrective Refocusing
Participants reexperience family conflicts in the group, which allows them to recognize and change behaviors that may be problematic

Group Phases
forming, storming, norming, performing, adjourning

Family Systems Therapy
Murray Bowen

-a person’s problematic bx may serve a function for the family or be a symptoms of dysfunctional patterns

KEY WORDS*

Self-Differentiation, Triangulation,

Triangles

Structural Family Therapy
Salvador Minuchin

-How, when, and who whom family members relate

KEY WORDS*

Mapping

Hierarchies

Boundaries

Strategic Therapy
Jay Haley

-Symptoms are a way to communicate metaphorically in a family

-Symptom focused

KEY WORDS*

Straightforward directive

Paradoxical directive (reverse psychology)

Reframing (you are not jealous of your sister you just care for her so much)

Solution Focused Therapy
-MIRACLE QUESTIONS

-EXCEPTION-BASED FINDING -SCALING QUESTIONS

Meditation
if teaching about meditation must tell them about MUSCLE RELAXATION

If patient tells you something BEFORE you ASSESS- EXPRESS EMPATHY “I’m sorry this happened to you.”

USE OPEN ENDED QUESTIONS unless talking to a child or someone that is unable to construct a narrative then use CLOSED ENDED or YES/NO

PICOT
P: Population
I: Intervention
C: Comparison
O: Outcome
T: Time

If a patient has rheumatoid arthritis check
ESR

Therapy session with husband and wife & only one shows up

Reschedule

A patient’s mother calls and tells you that her son has been sodomized by their 15 year old brother

tell mother DO NOT LEAVE THE CHILD ALONE WITH THE BROTHER

-Provider calls CPS

-Arrange crisis therapy for family

Patient is moving out of state

if there is no imminent danger then provide enough medication for them to establish a new provider

Level 1 evidence
systematic reviews of random control trials (RCTs) or Meta-analysis or RCT-highest internal validity due to randomizations
At least (2)

Level 2 Evidence
systematic reviews of cohort studies
-little bias because the subjects are identified prior to outcome – randomization is lost

Shrill Cry
Intracranial pressure

Child between the ages of 3-6 masterbating
Normal to play with genitals (PHALLIC STAGE) NORMAL do not assume they have been abused*

Mom is concerned that her son age 10 has swelling in his chest and she is concerned he is developing breasts
Young boys ages 9-16 years old often have NORMAL BREAST ENLARGEMENT which disappears within 6 months

*reassure them that this is normal

Elderly female presents with decreased sex drive
Check Testosterone level

Sex Hormone-Testosterone is involved in sex drive

-MUST KNOW THAT WOMAN have TESTOSTERONE TOO

alcohol dehydrogenase
an enzyme active in the stomach and the liver that metabolizes alcohol

-WOMAN HAVE LOWER ALCOHOL DEHYDROGENASE (metabolizes alcohol)

-This is why woman get drunk faster

-Lower levels of this enzyme may also cause a higher propensity to develop LIVER DISEASE

When OB wants to hire psych providers

they want to increase mental health access to those that need it the most

Normalizing grief and loss in children

Don’t tell them what to do because grief responses vary

-i.e Don’t tell them to stop working that is prescriptive advise

-With children the most important thing is to reinforce FAMILY support an supportive therapy such as group therapy so they can learn from other children who have experienced similar events

palmar grasp reflex
normal up to 5-6 months
If older baby still has this reflex->refer to specialist

Moro (startle) reflex
Normal till 5-6 months
If present past normal range->refer to specialist
If not present within the normal age->Xray may be a sign of a broken bone, nerve injury, or spinal injury

Babinski reflex
Normal up to 2 years
If present past normal range->refer to specialist

PDE-5 inhibitors
Sildenafil (Viagra)

Vardenafil (Levitra)

Tadalafil (Cialis)

RAPIDLY ABSORBED

Used for erectile disfunction

Difference between BMI in anorexia vs Bulemia
Anorexia-Low BMI
Bulemia-Normal BMI

Parent brings in 16-year-old with BMI 12, Pulse LOW, BP LOW and you determine the patient needs to be sent for medical evaluation but the parents refuse

Contact CPS

You read and article that says that most children with ADHD abuse substances…

-To translate this information into practice what should you do?

-Screen ALL children for ADHD for SUBSTANCE USE

-OR Screen ALL children with SUBSTANCE USE for ADHD

accupuncture
used for pain and depression

Habeas Corpus

legal concept that protects patients from unlawful hospitalization

-May be a reason to leave AMA

Disseminated Encephalomyelitis

inflammation of nervous system

-MUST DO NEURO EXAM

-EXAM: May present with ASSYMETRICAL BODY MOVEMENTS

Assume you are interviewing a patient and you want them to provide information in a specific timeline, but they are unable

TO help ask them specific questions which helps to ANCHOR their memory

Before you administer a medication, you must educate them about the medication but first you should assess

BUT FIRST ASSESS WHAT THEY KNOW ALREADY or WHAT THERE BELIEVES ARE ABOUT THE MEDICATION

Patient presents with iatrogenic effect
assess ALL the medications that the patient is taking

-Don’t assume that it is from the medication you prescribed

When trying to pass a policy and your co-workers are against it what should you do
educate them on how the policy will benefit patient care

To promote a policy how do you get the word out there
Think most FEASIBLE option with WIDE net or audiance

Working in outpatient setting and you want to ensure continuous improvement in quality of care.

-Create an instrument to monitor clinical outcomes (this helps to identify what you are doing right or wrong)

Autoimmune disease can lead to increased
Cytokine level

If a child is urinating the bed
-Teach parents to use alarm clock to wake up to urinate (NON PHARM FIRST)

-If that doesn’t work try DESMOSPRESSIN (decreased enuresis)

Are you allowed to look up a patient on social media?
No it violates their trust

Assume you started a patient on a medication and they go home and find out that there is a black box warning on the medication that you were unaware of. They call with concerns…

-First, go online and do your own research

-Research the RISK vs BENEFIT before you tell the patient to stop the medication

Risk factors for sleep apnea
excessive weight, obesity, diabetes, smoking

Tolerance
you need higher doses of the medication in order for the medication to be effective

PHQ-9

5 mild depression
10 moderate
15 moderately severe
20 severe

Max score 27

HAM-D

10 Mild
14 Moderate
17 Severe

HAM-A

8 Mild
15 Moderate
24 Severe

Beck Depression Inventory (BDI)
0-13 Subclinical

14 Mild
20 Moderate
29 Severe
40 EXTREME

Max score 63

GAD-7 Scoring
0-4: Minimal Anxiety

5 Mild Anxiety
10 Moderate Anxiety
15 Severe Anxiety

Max score 21

COWS
Medicate with PRNS at score of 7 or above

Consider Subutex or Suboxone at 13 or above

Remember Methadone is the LEAST safe option due to cardiac issues

CIWA
Begin PRN medication at 8 or above

Scores of 15 or above consider scheduled medications

Patient in alcohol withdrawal and you are choosing medication for CIWA, check what?
LFT

if liver disease use ATIVAN because of short half life over VALIUM

REMEMBER DETOX SHOULD NOT OCCUR OUTPATIENT–>residential or inpatient is needed especially if pregnant. REMEMBER SAFETY FIRST

Idealization
Seeing someone else as perfect, ideal, or more worthy than everyone else

This is often seen in grieving before acceptance of the loss

Appreciative Inquiry
is an approach to organizational change which focuses on strengths rather than on weakness

Example do not focus on what the employee does wrong focus on what they do well

Reflective Practice
Links theory to practice with a goal of correcting practices that are incorrect. Example Debriefing after a restraint to find out what went wrong or what went right DEBFRIEFING MAY BE A KEY WORD

-After an incident–> Debrief

Assume you started the patient on an antidepressant and now they complain of insomnia

  1. Before changing medication, CHANGE THE TIME OF DAY THEY ARE TAKING THE MED

Conflict of interest between pharmaceutical companies and Nurse Practitioners i.e promises to sponsor NP loan forgiveness. You want to create a policy to address this. First you must examine…

study the relationship between the industry provided samples and industry sponsored education

If a patient is involuntarily admitted can they still refuse medications?
they can still refuse medications, UNLESS it is an emergency or if the court determines they must take medications

Scope of practice is determined by
State board of Nursing

Scope of practice defines
NP roles and actions
-Varies broadly state to state

If you would like to perform ECT as an NP what should you review
The state scope of practice standards to see if it is allowed and what certification is needed

Standard of practice is determined by
ANA

-Provides a way to judge nature of the care provided

The PMHNP is required by law to carry out care in accordance with what other reasonably prudent nurses would do in the same or similar circumstances. Thus, provision of high-quality care consistent with established standards is critical

Exceptions to Confidentiality
-Answering court orders, subpoenas, or summonses *high yield (if you don’t release or lie about knowing this is PERJURY)

-Insurance companies

-Giving information to attorneys involved in litigation

-Intent to harm self or others

-Meeting state of federal requirements for reporting disease states

-When the need for information outweighs the principle of confidentiality i.e unconscious patient and their life is at stake

Tarasoff principle
1976 – duty to warn victims of potential harm from client

may vary by state so you must check with your state board of nursing first…it may not be your responsibility to notify

If you are taking care of a patient and during the process of interview she tells you that her husband just texted her that her husband is going to kill self

-Call the police, provide the address

informed consent
-Communication process between the provider and client that results in client’s acceptance or rejection of proposed treatment

-Ensure they understand the risks vs benefits

If patient is able to reiterate the risks vs benefits of procedure or treatment, they are able to give consent*

Example patient comes to the hospital, and they are unable to give consent i.e to sick to agree to treatment you must

assess need for involuntary treatment

Justice
doing what is fair, fairness in all aspects of care

Nonmaleficence
do no harm

Beneficence
doing good/promoting well-being

Fidelity
being true and loyal

Veracity
telling the truth, patients have the right to know the truth about their treatment

Autonomy
doing for self (right to self-determination)

New male patient has a 10 year history of substance abuse, depression, and anxiety. He is requesting Xanax. Which principle should the PMHNP employ moving forward?

A. Beneficence

B. Fidelity

C. Non-Maleficence

D. Veracity

C. Non-Maleficence

Do no harm, giving Xanax would endanger him secondary to the high abuse potential and imminent danger if he were to overdose on Xanax

If no history of substance use, then beneficence would be appropriate because the Xanax would help the anxiety

Acute agitation and anxiety vs acute agitation and psychosis

IM ordered
IM ativan for agit/anx

IM antipsychotic for agit/psychosis

Patients have the right to be treated in the Least
Restrictive Setting

The PMHNP is asked to consult with a local inpatient psychiatric facility to provide nursing staff development. After meeting with the administrator to identify the nature of the problem requiring the consultation, the PMHNP’s next step is to:
A. Create interdisciplinary teaching team
B. Develop Outcome measures
C. Market the educational plan
D. Utilize a survey to assess the educational needs of the staff
D. Utilize a survey to assess the educational needs of the staff

3 Interventions vs 1 Assessment FIRST YOU NEED TO ASSESS FIRST

What is the best way to reduce stigma
THROUGH EDUCATION

*THINK WIDEST AUDIENCE

Just Culture
individuals are continually learning, designing safety systems, and managing behavioral choices

The PMHNP is concerned about access-to-care issues in the local community and wants to help develop health care policy to help patients access care more effectively.
A. Asking the clinical manager to explore options for access
B. Organizing a political protest
C. Working with the local chapter of the nurses professional association
D. Writing letters to the editor of the local newspaper
C. Working with the local chapter of the nurses professional association

Remember strength in numbers but STAY INVOLVED i.e asking the manager just passes off

If a patient from a specific culture is refusing to accept any diagnosis of mental health disorders because of shame what could be done to address this barrier?

A. Educate the family

B. Political advocacy

C. Public Health Concern

D. Community education programs

D. Community education programs

*Narrow down to education A/D…then think WIDE NET =Community EDUCATION

A client with Biolar I disorder presents to your PMHNP office for a follow-up visit. During the visit the client informs you he no longer wants to be treated with medication. , and he does not have bipolar disorder, that was a misdiagnosis, He further informs you he stopped all his medications 2 months ago and is here to thank you for your care and tell you he no longer needs appointments. Understanding ethical conflict, you use which of the following ethical principles?
A. Autonomy
B. Nonmaleficence
C. Justice
D. Beneficence
A. Autonomy

Patient’s have the right to self-determination

Recovery Model * RELAPSE IS A LEARNING OPPORTUNITY

-Treatment approach that does not focus on full symptom resolution but emphasizes on resilience and control over problems in life

-Self-Direction (do not tell them what to do)

-Individualized and Person-Centered

-Non-Linear, Recovery is not a step-by-step process, but one based on continual growth, occasional setbacks, and learning from experience

In counseling a 23 y/o married Hispanic mother who brought her 4 year old son to the clinic for “mal de ojo” with symptoms of fitful sleep, diarrhea, vomiting, and fever the PMHNO;
A. Identifies what steps the mother has already tried in caring for the child
B. Explain that the symptoms are viral infection
C. Educates about importance of fluid electrolyte imbalance
D. Respects the mother’s understanding of the child’s illness
A. Respects the mother’s understanding of the child’s illness

*In cultural questions remember RESPECT FIRST! Even before assessment

Quality Improvement

Projects designed to improve systems, decrease cost, and improve productivity

What is an example of a quality improvement process?
Plan, Do, Study, Act

Retrospective Chart Review is an example of a Quality Improvement Process

If they ask HOW the NP would do a quality Improvement Process the answer may be Plan, Do, Study, Act

The NP is responsible for initiating quality improvement at a community clinic. The effective strategy for evaluating the clients services is to

A. Chart review analysis

B. A root cause analysis

C. Plan DO Study Act

D. Failure effect mode analysis

C. Plan Do Study Act

SBIRT
Screening, Brief Intervention, and Referral to Treatment

  • Use to screen substance use disorders

Erikson’s stages of psychosocial development

  1. trust vs. mistrust
  2. autonomy vs. shame and doubt
  3. initiative vs. guilt
  4. industry vs. inferiority
  5. identity vs. role confusion
  6. intimacy vs. isolation
  7. generativity vs. stagnation
  8. integrity vs. despair

Piaget’s stages of cognitive development

  1. sensorimotor
  2. preoperational
  3. concrete operational
  4. formal operational

Preoperational Stage includes
2-7 y.o- MAGICAL THINKING IS NORMAL, if they believe that monsters can fly this is NORMAL

Egocentric

Understand language

formal operational stage
12+ y/o during which people begin to think logically about abstract concepts

KEY WORD is LOGIC think like a scientist or do a science project you must be able to use logic ABSTRACT THINKING such as doing algebra

sensorimotor stage
in Piaget’s theory, the stage (from birth to about 2 years of age) during which infants know the world mostly in terms of their sensory impressions and motor activities
Object permanance

concrete operational stage
in Piaget’s theory, the stage of cognitive development (from about 6 or 7 to 11 years of age) during which children gain the mental operations that enable them to think logically about concrete events
i.e finding similarities in objects, grouping things

Risk factors for suicide
Sex (Male),
Age (Teenager or Elderly),
Depression,
Previous Attempt, Ethanol or Drug Use, Loss of rational thinking, Sickness (medical illness),
3 or more prescription medications,
Organized plan,
No spouse (divorced, widowed, or single especially if childless). Social support lacking.
WHITE

Women try more often. Men succeed more often.

A 72 year old female brought in by her husband with increasing forgetfulness, decreased activity, and decreased appetite for 2 months. She has a history of HTN and is being treated with Lisinopril. The exam is normal and the MMSE provides a score of 24 but she declines to answer some questions and needs to be urged to participate in the assessment. What is the likely diagnosis?

A. Alzheimer

B. Vascular Dementia

C. Depression

D. Medication Toxicity

C. Depression

*2 months=too soon for Alzheimer’s, 24 is mild MMSE, HTN is not enough info to diagnose vascular dementia HALLMARKS FOR VASCULAR DEMENTIA are carotid bruits fundoscopic abnormalities and enlarged cardiac chambers, remember PSEUDO dementia is DEPRESSION

HALLMARKS FOR VASCULAR DEMENTIA

carotid bruits fundoscopic abnormalities and enlarged cardiac chambers,

Patient comes to the office, and you score them on HAM-D a 23 and you start an antidepressant on dose Xmg, 2 weeks later they score a 16 on the HAM-D. What would you do?

Leave the dose where it is

Patient is taking Zoloft 200mg and on the GAD 7 they score a 2, what do you do

Leave the dose where it is

Zung Depression Scale Scoring

50 Mild
60 Moderate
70 Severe

25-49 is NORMAL RANGE
100 is max score

MMSE scoring
0-10 severe

10 moderate

20 mild

25 Normal

Kids under 10 years old are severely challenging to teach

HIGHER THE BETTER!

Teratogenic Effects

  1. Lithium
  2. Carbamazepine
  3. Depakote
  4. Benzo
  5. Epstein Anomaly
  6. Neural Tube
  7. Neural Tube (specifically spina bifida, atrial septal defect, cleft palate)
  8. Floppy Baby

Primary prevention
Efforts to prevent an injury or illness from ever occurring.
-Education
-Safety Initiatives
-Modifying environment

Secondary Prevention
Efforts to limit the effects of an injury or illness that you cannot completely prevent.
-Early findings
-Screening
-Prompt and effective treatment
Example: Crisis hotline, disaster response

Tertiary Prevention
-aims to prevent the long-term consequences of a chronic illness or disability and to support optimal functioning
-Rehab Services
-Day treatment
-Case management
-Social Skills training

Pharmacokinetics
what the body does to the drug

Pharmacodynamics
what the drug does to the body

messenger RNA codes for
amino acids

Poor relationships, lack of future hope, suspicious of others indicates developmental failure of what stage
infancy, trust vs mistrust

Poor self-esteem, low self control, self-doubt, lack of independence indicates failure of what stage
early childhood 1-3, autonomy vs shame and doubt

Lack of self-initiative, lack of goal orientation indicates failure of what stage
Late childhood 3-6 y/o initiative vs guilt

sense of inferiority, difficulty with working/learning indicates a failure of what stage
school age 6-12 y/o industry vs inferiority

identity confusion, poor self-identification in groups indicates failure of what stage
adolescence 12-20 y/o identity vs inferiority

emotional isolation, egocentrism indicates a failure of what stage
early adulthood 20-35 y/o intimacy vs isolation

self-absorption, inability to grow and change as a person, inability to care for others indicates a failure at what stage
middle adulthood 35-65 y/o generativity vs stagnation

bitterness, sense of dissatisfaction with life, despair over impending death indicates failure of what stage

65 y/o integrity vs despair

agonist effect
Drug binds to receptors and activates a biological response

Inverse agonist effect
Drug causes the opposite effect of agonist

partial agonist effect
Drug does not fully activate the receptors

Antagonist effect
Drug binds to the receptor but does not activate a biological response

Herbals that interact with warfarin
Vitamin E
Omega-3

Black Cohosh
herbal used for menopause

Bellandonna
herbal used for anxiety

chamomile
herbal used for sedation and anxiety

Ginko
Herbal used to treat memory, dementia, & sexual dysfunction from SSRIs

Ginseng
Herbal product used for stress reduction, fatigue, and depression

Valerian
herbal used for sedation

Hypertensive crisis can occur when MAOI are taken with
Meperidine
Decongestants
TCAs
Atypical Antipsychotics
St.Johns wart
L-Tryptophan
Stimulants
Asthma meds

Microcytic anemia
iron deficiency

macrocytic anemia
due to folate or vitamin B12 deficiency

Labs: Folic Acid, B12, ESR/CRP, HGB, MCV

14 y/o with no axillary hair and no period
Normal tanner stage, start by 16

Two classes of cardiac meds that should not be used together
ACES and ARBS (angiotensin receptor blocker)
Together can cause renal dysfunction

Abnormal Trendelenburg Test
Hip disease, refer child out, assessed during head to toe

too little acetylcholine
too much acetylcholine
Alzheimer’s
Parkinson’s and EPS

4 D’s Discover, Dream, Design, Destiny
Appreciative Inquiry

Cranial Nerve V
Trigeminal
Clenched Teeth

WBC 1500-2000
Biweekly labs
Less than 1000 Stop

How can you assess cranial nerve XII?
Ask patient to stick out their tongue

First sign of metabolic syndrome
large waist circumference

Grade 2/5 hoarse systolic heart murmur
aortic stenosis

Ibuprofen + lithium
increases the serum level of lithium up to double

Kleinfelter’s Syndrome
male with more than one X chromosome (XXY)
-Decreased sperm, fertility issues

Mental Health Parity Act
forbids health plans from placing lifetime or annual limits on mental health coverage that are less generous than those placed on medical or surgical benefits

what does nuchal rigidity indicate?
meningitis

Patient is on interferon and lexapro, as a provider you understand that
interferon can increase depression therefore you may have to increase the lexapro

Patient complains of neuropathic pain and neurontin is not working
Try Lyrica (pregabalin) its absorbed quickly and the maximum rate of absorption is 3x of Neurontin

Patient is in hospital with no family and is failing cognitive test what should you do?
MRI
Tox Screen

Phases of policy making
formulation, implementation, evaluation

Rhett Syndrome
a rare disorder found virtually exclusively in girls, is a neurodevelopmental disorder in which the child usually develops normally until about 6 to 18 months of age at which characteristics of the syndrome emerge; characteristics include: hypotonia (loss of muscle tone), reduced eye contact, decelerated head growth, and disinterest in play activities

Signs of fetal alcohol syndrome
small head, smooth palpebral fissure, inner epicanthal folds, thin upper lip

Tegretol side effects
Aplastic anemia, agranulocytosis, steven johnsons, hyponatremia. Watch with cipro and erythro

Telemedicine legal question?
Licensing Jurisdiction for the NP must be considered

A person is seen wandering the streets for 2 days
Delirium

Medication used for serotonin syndrome
Cyproheptadine

Why? it is an H1 blocker but is also has serotonin receptor blocking activity. Specifically, it acts to block 5-HT1A and 5-HT2A receptors which are the ones responsible for serotonin syndrome

When to assess a patient in restraints?
initially within 1 hour; then 8 hours

Why would you be concerned with immature reticulocytes?
Reticulocytes are involved in conditions affecting RBCs such as anemia.
-Low reticulocytes may be seen is iron def. anemia, pernicious anemia, folic acid deficiency, and aplastic anemia

pharm treatment for agoraphobia
short term benzo, SSRI, SNRI, TCA, or beta-blocker off label

Anorexia admission Criteria for hospitalization includes:
weight loss over 30% over 6 months
severe hypothermia temp less than 96.8
HR less than 40
BP less than 70
Hypokalemia less than 3mEq/L
BMI<16

ANOVA
ANalysis Of VAriance – btwn means of 3 or more groups
An inferential statistical test for comparing the means of three or more groups

precontemplation stage
stage of change in which people are unwilling to change their behavior

Contemplation stage
person is considering making a change, aware that there is a problem but is not quite committed to changing

Preparation Stage
Person has made the decision to change, is ready for action

Action Stage
Person is engaging in specific, overt actions to change

Maintanence stage
The person is engaging in behaviors to prevent relapse

These 3 meds cause BIG FREAKING PROBLEMS
Strong inhibitors of 2D6
Bupropion, Fluoxetine, Paxil

Boy tells you he wishes to be a girl and asks you not to tell the parents
Don’t tell

BRUISE on the padded part of his arms
Say I see you have bruises on your arm
may I Ask what happened

Can an advanced directive be revoked? How?
Yes
at any time

Can you take Buspar during pregnancy?
Category B – ok if really needed.

Carb and barb + Coumadin
Strong Inducers of 3A4 can decrease INR

Who is in charge of the DEA?
State and Federal

Common comorbidities of bipolar
anxiety, alcohol, substance use

Conjunctival injection, munchies, psychomotor slowness?
Marijuana intoxication

Diary Log
CBT

Depakote and Disulfiram
increases INR

What 3 atypicals can be used with teens?
Zyprexa, Abilify, Seroquel – low doses

  • also Risperdal

Np wants to implement a certain policy in nursing.
start with nurse manager

Phenycyclidine (PCP) can cause?
Nystagmus

Hildegard Peplau
Theory of Interpersonal Relations
Nurse as therapeutic tool
-Care for the person as well as the illness
-Patients are PEOPLE not DIAGNOSES

Patient acting out due to missing session what do you do?
You relate to childhood abandonment and talk about it with the patient

Patient on antidepressants for 3 weeks and attempted Suicide
stop the medication immediately

Patient shows symptoms of dizziness, tremors, sweating, What Medical Diagnosis
hypoglycemia

Pearson’s r
a statistic that measures the direction and strength of the linear relation between two variables that have been measured on an interval or ratio scale

pincer grasp
9 months

Problems in the parietal lobe can lead
Sensory-perceptual disturbances and agnosia(inability to perceive objects)
R-L confusion
Difficulty writing (agraphia)
Aphasia(difficulty of language)

To promote resilience in a patient with schizophrenia that lives alone consider referral to
-ACT
-Peer support

Patient on Lithium and Depakote and has temp, right flank pain, brown urine. What do you do FIRST?
Check LFT
If fine then check creatinine

Pt states “god did this to me.”
Assess spiritual needs first

Patient taking breathing treatment Albuterol/Proventil
Do not take MAOI or TCA

p-value
The probability of results of the experiment being attributed to chance.

Reliability
consistency of measurement

Stereogenesis
identify an object without sight
i.e dice in hand

Tagamet (cimetidine)
H2 receptor antagonist (antacid)
Increases benzo
Increases coumadin

Trazadone concerns
EKG-QT prolongation
Priapism
Glaucoma

Turner Syndrome
A chromosomal disorder in females in which either an X chromosome is missing, making the person XO instead of XX, or part of one X chromosome is deleted.
-Delayed puberty
-Amenorrhea
-Web neck, osteoporosis, lymphedema
-poor social skills

To start your own firm as an NP and need to examine economic viability
Show Revenue and expenses

3 CK muscle enzyme tests?
CKMM, CKBB, CKMB (normal 0.3 mmcg/L)

What are the legal ramifications of treating someone without informed consent?
The same as they are with informed consent
-Respect
-Beneficence
-Justice

Two important things to measure when prescribing Zyprexa
Waist circumference
Lipids

What schedule of controlled substances are NPs allowed to prescribe?
II-V

What crania nerve is affected when you ask the patient to shrug their shoulders?
XI (11) Spinal Accessory

These medications are renally metabolized
Gabapentin
Campral
Lithium

grapefruit juice
inhibitor that can reduce the absorption of the drug by 47% therefore the drug blood levels will be increased
-Decrease dose of drug

How do Asians see HC providers?
As in a position of authority.
Expect to give instructions and help make decisions

How do you protect from the evil eye?
Red ribbon on an infant
Amulet for adults

T-test
assesses whether the means of two groups are statistically different from each other

Treatment for children with panic disorder
clonidine
guanfacine

What do BCP’s do to Lamictal?
Inducer – will lower dose of Lamictal

What does an increased retic count indicate
Bone marrow disorder or Vitamin Deficiency
Normal Range 0.5-1.5

What do you see in Labs with HIV dementia
CD4 <200
Viral Load is high
<20% get it with antiretroviral treatment

What is occuring in the adolescent brain?
Dendritic pruning
Emotions are controlled by amygdala
Prefrontal Cortex is still not fully developed (may be why young boys are risk takers)

Indomethacin is a
NSAID- WATCH WITH LITHIUM

Tramadol
Highly serotonergic

Dissemination
the act of spreading widely
-publication-highest level
-Presenting at national conference
-Journal club

Sensitivity vs. Specificity
sensitivity – how well a test identifies truly ill people (True positive)
specificity – how well a test identifies truly well people (True negative)

*In medical diagnosis, testing sensitivity is the ability of a test to correctly identify those with the disease (true positive) whereas test specificity is the ability of the test to correctly identify those without the disease (true negative)

What is the purpose of HIPAA?
National standards for electronic HC transactions
-National ID for providers, health plans and employers.
-Not SIMPLY Confidentiality.

DETROL interactions
Topamax
KCL
Zonegran

Yale-Brown Obsessive Compulsive Scale (Y-BOCS)
OCD
0-7 subclinical
8-15 Mild
16-23 Moderate
24-31 Severe
32-40 Extreme

If you want to decrease the use of seclusion who would be considered the primary change agent
Unit staff

Rennie vs Klein

  • right to refuse any treatment
    Until court orders it
    “due process”

Roger vs. Oken
determined that patients have an absolute right to refuse treatment, but a guardian may authorize their treatment.

ROGERS GUARDIAN

Donalson vs O’Connor
Confinement
-It is unconstitutional to commit a person involuntarily who is not imminently dangerous to self or others

Donalson was a patient hospitalized for 15 years

Dusty vs United States
incompetent to stand trial

Durham vs King
Insanity defense

Riese vs St. Mary’s Hospital
7/8 8B ruling says that there should be court determination of incompetence for involuntary committed mental persons to receive antipsychotic medications

Stark Law
Prohibits physicians or their family members who own health care facilities from referring patients to those entities if the federal government, under Medicare or Medicaid, will pay for treatment.

There is a medication class that is contraindicated when a patient has tics. What is that class of medication?
Stimulants –often used for ADHD

What action do you take if a patient reports being a victim of abuse.
Safety first!
Provide patient with an environment of safety and reassure them of their safety in the clinic/hospital.

If you work inpatient, what do you do before discharging a patient who says they have been abused?
Make sure the patient will be safe.

If you work inpatient, is a safety contract sufficient to determine that you are releasing a patient into a safe home?
No. A safety contract is not enough. The NP has to confirm that the patient is going to be safe for themselves or their environment is safe.

Inpatient–for safety, where should the NP interview the patient and why?
Safety is more important than privacy. In an office with a door open or partially open. The day room would be a breach of confidentiality and a closed door or in their room would give the patient access to harming the NP.

If a child comes into the office with their parents and say they have been abused, what action does the NP take?
interview the child separately from the parents THEN report to CPS

If a child comes into an appointment without their parents and plays with a toy in a sexual way, what action does the NP take?
This makes the NP suspect abuse so initial action is to immediately report to CPS.

What is the cornerstone of building a therapeutic alliance with adolescents?
Confidentiality with the explanation that confidentiality must be broken if they are a danger to self or others

Do we interview adolescents with their parents in the room?
No because the adolescent may have info that they want kept confidential from their parents.

If an adolescent comes in with their parents, do you keep the parents and adolescent together or separate theme?
Separate them and speak to adolescent alone because the adolescent has a right to confidentiality.

Which ethnic group has the highest incidence of suicide and suicide attempts ?
Native American

Native Americans believe in their healing stick. If a staff member tries to take it away, what should the NP do?
Educate the staff member in cultural competency and sensitivity.

If a patient wants a traditional healer to accompany them to an appointment, what does the NP have to do ?
Get consent from the patient and allow the traditional healer to attend as well. With permission from the patient, you can include the healer in the care plan as follow up.

Which labs should be taken before treating for depression or mania?
TSH

What is the normal range of TSH?
0.5-5.0 mu/L

Symptoms of hypothyroidism mimic which mental illness?
Depression

Symptoms of hyperthyroidism mimic which mental illness?
Mania

If TSH is low i.e. 0.4 or lower, what is happening to T4 an T3 and what condition does patient have?
T4 and T3 will be high and patient will have hyperthyroidism

If TSH is high e.g. 7.0, what is happening to T4 an T3 and what condition does patient have?
T4 and T3 will be low and patient will have hypothyroidism.

What are the symptoms of hyperthyroidism that can mimic mania? And what are other symptoms?
agitation, anxiety, irritability, mood swings, weight loss.

Other symptoms are heat intolerance and tachycardia

What are symptoms of hypothyroidism that mimic depression? And what are other symptoms?
lethargy, weight gain, decreased libido.

And, cold intolerance

What is the Black Box warning for Depakote?
Pancreatitis

What are physical side effects/ dangers of Depakote (Divalproex)–for patient or fetus
Spina bifida in fetuses
Hepatotoxicity

If a patient taking depakote has signs of hepatotoxicity what would those signs and symptoms be ?
Abdominal pain in upper right quadrant of abdomen
Reddish brown urine
Yellowing of the skin and whites of eyes
Fatigue

If patient taking depakote has signs of hepatotoxicity, what should the NP do?
A liver function test –check AST and ALT levels

Signs of valproic acid toxicity?
Disorientation
lethargy
Respiratory depression
Nausea/vomiting

What action do we take at signs of valproic acid (Divalproex/Depakote) toxicity?
Dc medication and check depakote levels, do a LFT and check ammonia levels

What are the signs and symptoms of pancreatitis
upper adbominal pain
abdominal pain that radiates to patient’s back

tenderness when touching the abdomen
fever
rapid pulse
nausea
vomiting
oily stools

What in mental illness is Kava (or Kava Kava) used for ?
Kava Calms

anxiety
stress
insomnia

What is the major side effect of Kava? What do we monitor?
Liver damage
We monitor patient for RUQ pain and do LFTs

Kava is contraindicated with which meds/med classes?
Xanax (Alprazolam)
CNS depressants/ sedative medications

Benzos (CLonazepam/Klonopin), (Lorazepam/ Ativan),

phenobarbital (Donnatal)

Zolpidem (Ambien)

Lamictal and weight
Lamictal is the mood stabilizer that causes the least weight gain

Lamictal and rash
Can cause Stevens Johnson

What are the symptoms of Stevens Johnson syndrome?
Body aches, red rash, peeling skin, facial and tongue swelling

Which antipsychotics cause the least weight gain?
ZAL
Ziprasodone (Geodon)
Aripriprazole (Abilify)
Lurasidone (Latuda)

For patients taking antipsychotics that have caused weight gain, what routine labs do we check?
bmi
hip-to-waist ratio
glucose
lipid panel

Non-pharm treatment of antipsychotic induced weight gain (AIWG)–1st line
Exercise and nutritional counseling

This is first line treatment

Pharm intervention for Antipsychotic Induced Weight Gain
Switch to antipsychotic with lower potential for weight gain: ZAL

Black Box Warning for Carbamazepine
agranulocytosis, aplastic anemia, Stevens –Johnson syndrome

symptoms of Aplastic anemia
pallor
fatigue
HA
fever
nosebleeds
bleeding gums
skin rash
SOB

If prescribing Carbamazepine for an Asian patient, what gene do you screen for ?
HLA-B* 1502 allele is highly associated with Carbamazepine-induced Stevens-Johnson syndrome

ANC level that indicates NP should DC clozapine or Carbamazepine?
An ANC less than 1000 mm3 whether patient is showing signs of infection or not.

What are signs of infection for which we should monitor patients on Clozapine or Carbamazepine to DC it?
Sudden fever
Chills
sore throat
weakness

Lithium and neuroprotection
Lithium is neuroprotective treatment of choice for bipolar disorder– can protect nerve cells from damage.

What is the therapeutic range of Lithium
0.6-1.2 mEq/L

At what Lithium level does lithium toxicity occur?
1.5 mEq/L or higher

Lithium is the gold standard for treating what?
Mania

Lithium and suicide.
Lithium is the only mood stabilizer with evidence of anti-suicidal effects in bipolar

What is the therapeutic range of depakote?
50-125 micrograms (ug/ml)

What is the toxic range of depakote(valproic acid)?
150 microgrms (ug/ml)

Necessary labs for Lithium
TSH
Serum creatinine
BUN
Urinalysis
HCG for females age 12-51

Signs of lithium toxicity
In mild lithium toxicity, symptoms include WATCAD–Weakness, Ataxia, Tremor, Concentration poorness and Diarrhea.

weakness, worsening tremor, mild ataxia, poor concentration and diarrhea.

With worsening toxicity, vomiting, the development of a gross tremor, slurred speech, confusion and lethargy emerge

When to DC Lithium
serum level of 1.3 or 1.4 and signs of Lithium toxicity.

with or without symptoms if 1.5 serum level –toxicity level

If creatinine or BUN are high because Lithium is processed through kidney

Preventative tests when prescribing antipsychotics to women
Take HCG test/ aka pregnancy test before placing any woman on antipsychotic -for females of age 12-51.

If patient has 4+ protein in urine while on lithium what do we do ?
Monitor closely for lithium toxicity

What are side effects of Lithium?
hypothyroidism, ebstein anomaly, and nephrogenic diabetes insipidus, fine hand tremors, Maculopapular rash, GI upset(Diarrhea, vomiting, cramps, anorexia), polyuria, polydispsia, T-wave inversions, Leukocytosis (increased WBCs)

What do we do in cases of Lithium toxicity?
DC Li and check serum Li levels

What are some factors that can increase Lithium levels?
Kidney disease or drugs that reduce renal clearance
NSAIDS(ibuprofen, Indocin)
Thiazides (hydrochlorothiazide)
ACE inhibitors
Medications used for cardiac failure (lisinopril)
Dehydration
Hyponatremia (low sodium levels)

What causes Neuroleptic Malignant Syndrome (NMS)?
Antipsychotics

What are the symptoms of Neuroleptic Malignant syndrome (NMS)?
Extreme musclular rigidity
Mutism

Elevated CPK (happens due to muscle contraction and destruction)

Myoglobinuria
Elevated WBCs(leukocytosis)
Elevated LFTs

What is the treatment for NMS?
DC the antipsychotic and treat with:

There are two with different MOAs:

Bromocriptine (Parlodel) which is a Dopamine (2) agonist

Dantrolene which is a Muscle Relaxant

What are the signs of both NMS and serotonin syndrome?
Hyperthermia
Tachycardia
Diaphoresis
Altered level of consciousness

What are the signs of Serotonin Syndrome?
Hyperreflexia
Myoclonic jerks

What causes serotonin syndrome?
SSRIs/SNRIs/TCAs/MAOIs

How do we treat Serotonin Syndrome?
DC the offending agent and treat with

Cyproheptadine

When switching from an SSRI to an MAOI, how long wait before starting the MAOI?
wait 14 days

When switching from fluoxetine (Prozac) to MAOIs how long to wait before starting the MAOI?
5-6 weeks

When switching from an MAOI to Prozac how long wait until start Prozac?
wait 2 weeks

Why the wait times for switching from between SSRIs and MAOIs?
need a washout period of 5 half-lives between cessation of previous drug and introduction of new drug. This is the time it takes for the medication to degenerate out of the system

Due to risk of Serotonin Syndrome, which combination of meds to we avoid?
Combinations of SSRIs and SNRIs or TCAs or MAOIs or St. John’s Wort

If we see “regenerate” in answers related to serotonin, what do you do ?

eliminate. “Regenrate” is wrong answer

Why are SSRIs the 1st line antidepressants used in depression
because they are safer in cases of overdose

Serotonin Syndrome and triptans?
Triptans can cause serotonin syndrome so no triptans (for migraines) or sumatriptan (imitrex)

Which antidepressant to do we give a patient who is depressed and has cancer?
Citalopram or escitalopram because lower incidence of drug-drug interaction.

Which antidepressant to do we give a patient who is depressed and has neuropathic pain?
SNRI or TCA
An SNRI is safer

What do we prescribe for patients in chronic neuropathic pain?
Alpha 2 Delta Ligands= Gabapentin and Pregabalin (Lyrica)

antidepressants and sexual dysfunction
SSRIs/SNRIs can cause sexual dysfunction

Important info about Wellbutrin
NDRIs do not cause sexual dysfunction = Wellbutrin

Patient depressed +Low energy +fatigue give Wellbutrin

Wellbutrin contraindicated in hx of seizures or eating d/o = Wellbutrin increase seizure risk

Black Box Warning on all depressants and responsibiltity of NP
increase thoughts of self-harm in adolescents.

assess for frequency and severity of these thoughts at every visit.

Alcohol and depression
If patient is depressed, assess alcohol intake because some patients use alcohol to self-medicate and this can become a barrier to treatment

Prozac and Insomnia
Prozac can cause insomnia; encourage pt to take Prozac in the morning

Mental illnesses that cause thoughts of self-harm
Depression
bipolar
alcohol abuse
eating disorder
schizophrenia

mental illness most often associated with Homicidal ideation
Antisocial personality disorder

Placebo rate, children, antidepressants
few experimental studies exist that investigate the placebo effect of antidepressants in children and adolescent (When compared to adults, children with depression have a reduced placebo representation in studies but a higher placebo rate).

positive symptoms of schizophrenia
•Hallucination, Delusion, Loose association
•Ideas of reference (paranoia/ paranoid delusions)
•Agitated and bizarre behavior

Negative symptoms of schizophrenia
•Avolition – Asociality (Hygiene, Work)
•Anhedonia – Asociality (Interest, Relationship)
•Blunted affect
•Paucity of thought

age of onset of schizophrenia
18-25 males
25-35 in females

What is going on in brain of a schizophrenic that we see in an MRI/PET scan?
Ventricular enlargement

Everything else is decreasing in size
–Different lobes
–Different parts of limbic system
–Cerebral blood flow

What are the parts of the brain in which abnormalities or changes/deficits cause agression, impulsitivity, and abstract thinking problems in schizophrenia?
prefrontal cortex
amygdala
basal ganglia
hippocampus
limbic regions

Medications for which schizophrenics have a low tolerability so they are not as neuroprotective for schizophrenics as they are for others
Alpha 2 adrenergic receptor agonist (guanfacine and clonidine)

Why should we not give stimulants to schizophrenics?
Because stimulants can potentiate dopamine release

Non-pharm management of schizophrenia
manualized group therapy and assertive community treatment (ACT)

What is ACT?
ACT is a form of rehabilitation post-hospitalization

If a schizophrenic has a long-term history of non-adherence what may they need for in home care?
Referral to case management team so a nurse can go to their home and administer their medication. And, referral to aerobic exercise program

What level of care is social skills training for a schizophrenic?
Tertiary

How does exercise help schizophrenics?
Improve cognition
Improve quality of life
Improve long term health

A schizophrenic is taking oral Haldol and is at high risk of relapse (multiple hospitalizations), what should we do about administering medication
switch to intramuscular Haldol

Dosing for switching from oral Haldol to Haldol Decanoate
20 X the total daily oral doses=Decanoate dose–example:

5 mg PO BID
LAI–20 X 10 mg = 200 mg

What is the dose limit of LAI Haldol that can be given in one week?
100 mg
If they need 200 mg then give 100 mg on day one and come back in 5-7 days for another 100 mg;

same for 300 mg= 100 mg day one
5-7 days for second 100 mg
and 5-7 days for third 100 mg

What is a delusion?
Firm belief maintained despite evidence to the contrary.

If doing a MSE of preschooler(3-5y/o), what is the most important approach to
listen and observe cues—depends on clinical observation

Components of a Mental Status Exam–what do you assess with thought process assessment?
Assess the organization of the patient’s thoughts and ideas.

Components of a Mental Status Exam–what do you assess with thought content assessment?
Refers to the themes that occupy the patient’s thoughts and perceptual disturbances. Ex:
Suicidal ideations
homicidal ideations
SI or HI plan
visual hallucinations
auditory hallucinations

Mental status exam–when evaluating thought process or thought content, what are we evaluating? Are we evaluating organization of speech?
We are evaluting thoughts and ideas

NO, we are not evaluating organization of speech

The Folstein Mini Mental Status Examination is used with which population and for what ?
Adults–to assess cognitive decline

What are some important components of the mini-mental status examination
Concentration/attention/ calculation–examples: count backward from 100 by 7s

Orientation: Year, season, date, day, month, country, town, hospital, floor

Registration/ability to learn new material: say names of three unrelated objects clearly and slolwy, ask patient to repeat immediately

Recall (memory): Ask patient if they can recall the three object words previously asked to remember 5 minutes after introducing the object words

Fund of knowledge: Who is president/govenor

What is the purpose of the Clock drawing test ?
Very quick way to screen for possible dementia– takes 1-2 minutes to complete

If there are impairments on the CDT (Clock Drawing Test), which part of the brain may be damaged?
right parietal lobe i.e. the right hemisphere of the brains

What makes an antipsychotic atypical?
Atypical antipsychotics have both dopamine and 5HT2A receptor antagonism

Lower chance of EPS syndromes

Which type of antipsychotic for first psychotic episode?
Atypical antipsychotic that can be administered IM like Invegga, Geodon or Abilify.

Dopamine Pathway: Mesocortical pathway relationship between dopamine and schizophrenic symptoms
Meso–not major depression but meso depression + other negative symptoms
decreased dopamine in the mesocortical pathway is thought to be responsible for negative and depressive symptoms of schizophrenia

Nigrostriatal pathway of brain controls what ?
Nigrostriatal pathway mediates motor movements

Dopamine blockade in this pathway can lead to increase acetylcholine levels–increase salivation, teary eyes, diarrhea

Dopamine Pathway: Nigrostriatal pathway relationship between dopamine and EPS
Blockade of dopamine receptors in the nigrostriatal pathways can lead to EPs e.g. acute dystonia, parkinsonism, and akathisia

Long standing D2 blockade in the nigrostriatal pathway can lead to Tardive Dyskinesia.

Neurotransmitters and EPS
Increased acetylcholine and decreased dopamine can cause EPS’

EPS and metoclopramide
Metoclopramide (Reglan) can cause EPS like Tardive dyskinesia and parkinsonism

How do you treat EPS except Tardive Dyskinesia?
Benztropine

How do we treat Tardive Dyskinesia?
stop/decrease meds, switch to new anti psych that doesn’t cause tardive dyskinesia (possibly clozapine)

Dopamine Pathway: Tuberoinfundibular pathway relationship between dopamine and physical symptoms
Blockade of D2 receptors in this pathway can lead to increased prolactin levels leading to hyperprolactinemia which clinically manifests as amenorrhea, galactorrhea (Risperidone), sexual dysfunction, and gynecomastia.

Long term hyperprolactinemia can be associated with osteoporosis.

Normal prolactin levels
Men–less than 20 ng/ml
Women — less than 25 ng/ml

impact of Cytocrome P450 enzyme CYP1A2 inducers on drugs metabolized on the pathway
decrease serum levels of drugs that are substrates of CYP1A2 enzymes which causes subtherapeutic drug levels

impact of Cytocrome P450 enzyme CYP1A2 inhibitors on drugs metabolized on the pathway
Increase serum levels possibly causing toxic levels of drug

Olanzapine (Zyprexa) and Clozapine are both metabolized on which enzyme?
Cytochrome P450 Enzyme CYP1A2

If a patient on olanzapine or clozapine stops smoking what do we do with the dose
Decrease the dose because smoking is an inducer and it is no longer decreasing the serum levels of the drug.

Which are more worrisome? Inhibitors or Inducers?
Inhibitors because they can cause toxic levels of a drug in the body.

Is Tegretol(carbamazapine) an Inducer or Inhibitor ?
Inducer

Are clarithromycin and erithromycin inducers or inhibitors?
Inhibitors

If a patient is taking tegretol (carbamazapine) and clarithromycin, tegretol an inducer and clarithromycin an inhibitor, what should an NP do?
Decrease the tegretol (carbamazapine) to avoid the inhibitors causing a toxic level of tegretol in the blood

Medications that cause mania
Steroids
Disulfram (Antabuse)
Isoniazid(INH)
Antidepressants in persons with bipolar

Medications that cause depression
Steroids
Beta blockeres
Interferon
Isotretinoin (Accutane)
Some retroviral drugs
Antineoplastic drugs
benzodiazepines
progesterone

Steroids and psychosis
Steroids can cause psychosis

Are flonase and prednisone steroids?
Yes

When taking medications that cause mania and depression, how do you dose the antidepresant or antipsychotic?
Increase the dose.

Neurotransmitters involved in addiction
Dopamine and GABA

What causes pain in anorexics after eating?
Delayed gastric emptying

medications that can delay gastric emptying
Ranitidine–Antihistamine and Antacid
Famotidine–Antihistamine and Antacid
Omeprazole–proton pump inhibitor

If a med decreases the absorption of psychotropic medications, what is the best way to take it?
Take 2 hours apart

Some medications that decrease absorption of psychotropics?
Antacids
PPIs like Protonix, Omeprazole

If a patient aged 65+ is given SSRIs, for what should we monitor them?
monitor for increased anxiety

What is a paradoxical effect of a medication?
when medication causes opposite effect for which is was prescribed. Avoid giving in the future

Heart issues and Geodon and Citalopram
Geodon and Citalopram can cause QT prolongation

What is the maximum recommended dosage per day for Citalopram?
40 mg
for those 65+– 20 mg

What is the greatest risk factor for bipolar
multigenerational bipolar

If a person’s first bipolar episode is after the age of 45, what is the likely cause?
A medical condition like a stroke

What are the symptoms of Mania?
Distractability
Insomnia
Grandiosity
Flight of ideas
Agitation/Activity increase
Sexual indiscretions (or other pleasurable activity)
Talkativeness
DIG FAST

What is apoptosis
programmed cell death

Primary symptoms of Borderline Personality Disorder
self-harming behavior
recurrent suicidal behavior

What is the only treatment for Borderline Personality Disorder?
DBT–decreases suicidality in Borderlines

Who created DBT?
Marsha Linehan

What is the diagnostic process for borderline
use their journals/diaries to help diagnose borderlines

Medication for a pateint with borderline presenting with irritability, anger and self-harming behavior
Lithium

Specific Medication for a patient with borderline presenting with depressed mood, emotional lability, interpersonal problems, rejection sensitivity, aggresion, hostility
Depakote

What is conversion disorder?
a mental condition–usually begining suddenly after a stressful experience– in which a person has blindness, paralysis, or other nervous system (neurologic) symptoms that cannot be explained by medical evaluation

Treatment is therapy

What is an adjustment disorder?
An unhealthy or excessive response to an event or change within 3 months of it happening

What are symptoms of adjustment disorder
with depressed mood — feelings of sadness, decreased interest, sleep disturbance, appetite changes

with mixed disturbance of emotion and conduct — A child has a mix of symptoms from anxiety, depression and conduct dysfunction–possible peer conflict, verbal altercations, insomnia, frequent crying

Differentiation between Adjustment disorder and MDD
Adjustment disorder has a specific cause.

How do we treat Oppositional Defiant Disorder?
Family therapy with emphasis on child management skills; teaching parents about positive reinforcement and boundary settings.

Child and parent problem-solving skills training

In ODD therapy, if no parenting skills are developed, what can ODD turn into
Conduct Disorder

What are key symptoms of conduct disorder
Aggression toward people or animals and property.

Lack of remorse for ill deeds done

Pharmacological treatment for conduct disorder
Targets mood and aggression–treat with antipsychotics, mood stabilizers, SSRIs and alpha agonists (Clonidine and guanfacine)

Diagnosing Tourette Syndrome
At least 2 motor tics and at least 1 vocal tic have been present, not necessarily at the same time–for more than a year

Tics are not caused by using a subtance or other medical condition

When children have motor tics are they rare and permanent ?
No. Children’s tics are common and often temporary

Primary neurotransmitter involved in Tourette Syndrome
Dopamine, Norepinephrine, serotonin (DNS)

Hyperactivity of Dopaminergic systems in brain can lead to Tourette’s

Pharmacological treatment of Tourette Syndrome
Clonidine (Catapres or Kapvay) or Guanfacine (Intuniv)

What is acute stress disorder?
Psychiatric diagnosis that may occur in patients within 4 weeks after a traumatic event Features include anxiety, insomnia, re-experiencing, avoidance behaviors–basically PTSD symptoms but duration of symptoms is less than a month. (PTSD has to last at least one month)

What are hallmark symptoms of PTSD?
Intrusive re-experiencing
Increased arousal (hyperarousal)
Avoidance of stimuli associated with trauma

Pharmacological management of PTSD
SSRIs, TCAs, Prazosin for nightmares

Non-Pharmacological management of PTSD
EMDR (preferred over CBT)
CBT

What are the phases of EMDR?
Desensitization phase
Installation phase
body scan phase

Which parts of the brain are affected in ADHD?
Frontal cortex
Basal ganglia
Abnormalities in prefrontal cortex–inattentive
Abnormalities of reticular activating system

ADHD — age amphetamines are approved for
children age 3 to adult

Heart and ADHD meds
Assess cardiac hx before beginning stimulants.

There can be elevated heart rate and BP; increase risk of heart attack and stroke

ADHD — age methylphenidate are approved for
children age 6 to adult

ADHD– what does the dorsolateral prefrontal cortex control?
Executive function
Cognitive process such as planning, working memory
Problem solving
How to direct and maintain attention to a task

Signs of stimulant abuse
Insomnia
Tremors
Increased BP
Heart palpitations

ADHD–If patient starts having symptoms again during the day, what does that indicate; what should be done?
Medication has been cleared by the body; consider an extended release dose.

If parent is anxious or scared of child starting stimulants what should be done?
address their anxiety—give them some support before continuing with psychoedcation

What is OCD?
Presence of anxiety-provoking obsessions (recurrent and persistent thoughts, impulses, or images) or compulsions (for example motor tics) that function to reduce the person’s subjective anxiety level

Which autoimmune illness should be considered with sudden onset OCD symptoms in children?
PANDAS–Pediatric Autoimmune Neuropsychiatric Disorders Associated with Streptococcal Infections.

Differentiate OCD from Tourettes
Tourettes= tics only

OCD = intrusive/ persistent thoughts and tics

Factitious disorder
Condition in which a person presents with physical or mental illness symptoms that are induced.

Malingering
symptoms are faked for secodary gain such as avoiding work or prison

factitious disorder imposed on another
a condition in which one person induces illness symptoms in someone else

Reactive Attachment Disorder
product of severely dysfunctional early relationships between principle caregiver and child —- results when caregiver disregards child’s physical/emotional needs–> behavioral/interpersonal problems later in life (fearful/inhibited/withdrawn/apathetic/shows no emotion towards caregivers/ disruptive/disorganized)

GAD (general anxiety disorder)
excessive worry for at least 6 months

Panic Attack
surge of intense fear or discomfort that reaches a peak within minutes as well as a sense of impending doom

Panic Disorder
diagnosis given when patient experiences recurrent panic attacks without apparent triggers

SSRIs for treatment of panic disorder
fluoxetine
Paroxetine
Sertraline
Venlafaxine

Definition and Symptoms of DMDD (disruptive mood dysregulation disorder)
Childhood (<18) depressive disorder that includes
Chronic dysregulated mood (“moody”)
Frequent intense temper outburts/temper tantrums
Severe irritability
Anger

Treatment for panic attacks
Betablocker like propranolol.

Contraindications for propranolol
Can cause bronchospasms so contraindicated in patients using bronchodilators like albuterol

Anorexia Nervosa symptoms
Low BMI (<15)
Amenorrhea
Emaciation
Bradycardia
Hypotension

Action if Anorexic with BMI less than 15?
Refer for hospitalization. If parent refuses, report to CPS

BMI of bulimia nervosa
Usually in normal range

What is the non-pharm treatment for Oppositional Defiant Disorder?

  1. Family therapy, with emphasis on child management skills;
  2. teaching parents about positive reinforcement and boundary settings
  3. Child and parent problem-solving training

If no parenting skills are developed what can ODD develop into?
CD-conduct disorder

What are the primary symptoms of Conduct disorder
Aggression towards human and animals and lack of remorse

What is the pharm treatment for conduct disorder?
Meds that target mood and aggression:

  1. antipsychotics
  2. mood stabilizers
  3. SSRIs
  4. alpha agonists (Clonidine and guanfacine)

What are the defining symptoms of Tourette Syndrome/Disorder

  1. At least 2 motor tics and at least 1 vocal tic
  2. Tics are not caused by a substance or other medical conditions

What is the expected permanence of children’s motor tics?
They are fairly common and can be temporary.

What are the primary neurotransmitters involved in tourette’s syndrome?
DNS= Dopamine, Norepinephrine, Serotonin

Hyperactivity of Dopaminergic systems in the brain can lead to Tourette’s

What is the primary pharm treatment for Tourette’s
Clonidine (Catapres, Kapvay)
Guanfacine (Intuniv)

What are secondary pharm treatments for Tourette’s ?
Atypical Antipsychotic
Haldol, Pimozide, Aripiprazole

DSM-5 of Acute Stress Disorder
Acute stress disorder occurs within 4 weeks of traumatic event.

Features include anxiety, insomnia, poor concentration, intense fear or helplessness, re-experiencing the event and avoidance behaviors–

It presents as PTSD but the onset of symptoms is less than PTSD

What is the minimum length of time that PTSD has to last?
One month

Hallmark symptoms of PTSD
Intrusive re-experiencing
Increased arousal
Avoidance of stimuli associated with the trumatic event

What is the pharm management of PTSD?
Prazosin for nightmares
SSRIs
TCAs

What is primary important non-pharm treatment for PTSD?
EMDR

What are the different Non-pharm treatments for PTSD?
EMDR

CBT

What are the parts of the brain involved in ADHD?
BAFaP

Basal ganglia

Abnormalities in reticular activating system

Frontal cortex

abnormalities in the Prefrontal cortex–inattentive type

Cardiology and ADHD stimulant
Assess cardiac history before placing patient on stimulants as they can cause elevated heart rate and bP and increase risk of Heart attack and stroke

If history or family hx of cardiac issues, get an ECG before starting.

What are the ages for amphetamines?
3 and up

what are the ages for methylphenidate?
Ages 6 and older

What are the ages for Alpha agonist or alpha 2 adrenergic receptors agonist?
Age 6 and up

What are the ages for Strattera?
Ages 6 and up

Which aspect of ADHD does the Dorsolateral prefrontal cortex control?
Executive function

Signs of Stimulant Abuse
Insomnia
tremors
increased blood pressure and HR
Heart palpitations

If a patient is having ADHD symptoms during the day after having taken their stiumlant, what does that indicate and what should we do?
Indicates the medication has been cleared by the body

We should consider an extended release

What to do if parent is really anxious or scared about Medication for their kid?
Address their anxiety–support them then continue with psychoeducation

Differentiate OCD from Tourettes
OCD–intrusive/persistent thoughts and tics

Tourettes: Tics only

Symptoms of OCD
presence of anxiety provoking obsessions–recurrent and persistent thoughts, impulses, or images or compulsions for example motor tics that funtion to reduce the person’s subjective anxiety level

What does PANDAS stand for and which mental illness’ acute onset may be an indication of having PANDAS?
Should be considered in all children with sudden onset OCD symptoms

Pediatric Autoimmune Neuropsychiatric Disorders Associated with Streptococcal Infections

Malingering
Deliberate faking of a physical or psychological disorder motivated by secondary gain.

factitious disorder
Condition in which a person does things to make themselves sick when they are not organically sick

Reactive Attachment Disorder
in children, a pattern of inhibited, withdrawn, apathetic behavior toward adult caregivers and show no emotion towards caregivers.
The product of a severely dysfuntional early relationship between the principal caregiver and the child

GAD
General Anxiety Disorder:
excessive worry for at least 6 months

Panic Attack
Abrupt surge of intense fear of discomfort that reaches a peak within minutes; a variety of psychological and physical symptoms including a sense of impending doom

Panic Disorder
Patient experiences recurrent unexpected panic attacks

Treatment for Panic disorder
Fluoxetine
Paroxetine
sertraline

SNRI–venlafaxine

Beta Blocker–this manages the elevated BP that occurs with a panic attack

Can also give benzos which aren’t as safe

Beta blockers can cause bronchospasms so they are contraindicated with
bronchodilators like albuterol

DMDD–Disruptive mood dysregulation disorder
Childhood depressive and mood disorder–diagnosis age 6-17

Chronic dysregulated mood
Frequent intense temper outbursts/ temper tantrums

Hallmark symptoms of Anorexia Nervosa
Low BMI (15 or less than 15)

Amenorrhea
Emaciation
Bradycardia
Hypotension

BMI for Bulimia Nervosa
BMI usually in normal range

if a patient has an irritable, depressed, labile mood, what is the first thing we should do ?
Administer the mood questionnaire

When should we assess for nightmare disorder?
if pt reports a nightmare and parents or other family members have nightmares

Which neurotransmitters are affected in autism?
Glutamate, GABA, Serotonin

What are the symptoms of autism?
Persistent deficits in social communication and social interaction
No response when called by name
Nonverbal communication
Little or no eye contact
often like to line up, stack, or organize objects and toys in long, tidy rows

Broken mirror theory of autism
Claims that dysfunction of the mirror neuron system may be the cause of poor social interaction and cognition

If delirium is caused by ETOH or substance abuse what treatment can we give patient?
Benzos

What are often the causes of delirium in older people and what should we test them for?
Infections
We should do a urinalysis with culture and sensitivity

Dementia and which vitamins levels should be checked
Progressive mental decline; personality changes occur, irritability
Check Vit B12 and Folic Acid levels

Differentiating between depression and dementia
with depression it is acute onset of memory problems like happening over 5 months.

Also, in combination with memory issues, they have depression issues.
“I do not know” responses are commonly depression. In dementia, patient often confabulates answers–will not say “I do not know”

Dementia memory decline happens over time i.e. over 12 months or longer

Pseudodementia
Primary diagnosis is depression
acute onset of memory problems like happening over 5 months

Which parts of the brain are involved in Dementia
subcortical–motor symptoms: lack of coordination, tremors, ataxia, dystonia

Cortical: Language (aphasia) and memory impairments

Early signs of HIV dementia
Cognitive decline
motor abnormalities
behavioral abnormalities

Lewy Body Disease symptom of importance for the test
Presents with recurrent visual hallucinations

Pick’s disease
AKA frontotemporal dementia/frontal lobe dementia

Personality, behavioral, and language changes (slurred) in early stage

What meds treat HIV dementia?
Antiretrovirals

if patient has history of high risk behavior and HIV dementia symptoms, what should we do?
give them an HIV test

Pharm treatment of psychosis and agitation in dementia?
atypical antipsychotics

What does the amygdala control?
aggression
fear
anxiety
emotions

What does the Hippocampus control?
emotions
stress
learning
memory

What does dopamine produced in the substantia nigra regulate?
motor movements

If you are taking care of a terminal infant what do you do with the parents?
give infant to parents and allow them to grieve for their loss

How should we speak of and refer about grief to children
Normalize grief and loss in children—psychoeducation on grief responses; group therapy
Grief responses vary so do not tell a patient or family how they should grieve

What is the most important factor in children’s healing from grief?
an intact family so they can adapt easily

mood disorder neurotransmitters
DNS (dopamine, nor, serotonin+ GABA+ Glutamate )

What are most important risk factors for osteoporosis?
Smoking
Caffeine
Lack of weight bearing exercises
Lack of dietary calcium and vitamin D

What drugs should be avoided with Kava Kava?
Alprazolam
CNS depressants (e.g., phenobarbital, zolpidem)

What is the normal range for TSH?
0.5 – 5.0 mu/L

What are lab values in Hyperthyroidism
Decreased TSH
Increased T4 and T3

What are labs in Hypothyroidism?
Increased TSH
Decreased T4 and T3

What are 5 primary symptoms of Hyperthyroidism?
Heat intolerance
Agitation, Anxiety, Irritability
Tachycardia
Mood swings
Weight loss

What are 4 primary symptoms of Hypothyroidism?
Cold intolerance
Lethargy
Weight gain
Decreased libido

Hypothyroidism mimics symptoms of what psychiatric disorder?
Depression

Hyperthyroidism mimics symptoms of what psychiatric disorder?
Mania

Valproic acid produces what teratogenic effect?
Spina bifida (neural tube defect)

What is the primary organ where valproic acid is toxic?
Liver (hepatoxicity)

What are 4 signs of hepatoxicity?
Abdominal pain in the URQ

Reddish-brown urine

Yellowing of the skin and sclera

Fatigue

What is the therapeutic range for valproic acid?
50-125 mcg/ml

What is a toxic level of valproic acid?

150 mcg/ml

What are 4 signs of valproic acid toxicity?
Disorientation
Lethargy
Respiratory depression
Nausea/vomiting

A pt on Depakote C/O Abdominal pain in the URQ and Reddish-brown urine. What do you suspect? What do you do?
Hepatotoxicity; hold med, draw LFT’s (priority), DVP level

With suspected toxicity with valproic acid, what steps/labs should be drawn?
D/C drug
Check drug level
Obtain LFTs
Obtain ammonia level

A pregnant pt is taking accutane, zyprexa and abilify; what med needs to be d/c’d?
Accutane (used for acne; Causes birth Defects!!!)

What herbal supplement is used for anxiety, stress, and insomnia? What is the major side effect with Kava Kava?
Kava Kava
liver damage

What are 5 symptoms of Stephen’s Johnson Syndrome?
Fever -Key sx to suspect SJS; high yield (HY)

burning of mouth/eyes -HY

Body aches

Severe red rash

Peeling skin

Facial and tongue swelling

Which mood stabilizer causes the least weight gain?

Lamotrigine (Lamictal)

Which 3 SGAs cause the least weight gain?

Lurasidone (Latuda)
Aripiprazole (Abilify)
Ziprasidone (Geodon)

Which SGA is the least sedating?
Aripiprazole (Abilify)

Your pt is taking Kava and you need to add an AS to manage their SCZ, which do you add?
Aripiprazole (least sedating)

What are 2 hallmark symptoms of delirium?
Acute onset

disturbance of LOC

impaired cognition

inattention

How are agitation and psychotic symptoms treated in delirium?
Low dose Haldol (QTc check first)

16 y/o C/O feeling funny after inhaling Acetone (nail polish remover). What is your action?
Do UDS; chances are they may be abusing other substances

What 4 drugs can cause mania?
“SAID”

Steroids

Antidepressants (in BPD)

Isoniazid (INH)

Disulfiram (Antabuse)

What 6 drugs can induce depression?
“SIP BARB”

Steroids

Interferon,

Progesterone

Beta blockers

Accutane, (Isotretinoin)

Retroviral drugs, Antineoplastic drugs

Benzodiazepines

Steroids can induce what 3 psychiatric states?
Mania
Depression
Psychosis

REMEMBER SAFETY FIRST; pt s/p assault, 1st thing you do is_____.
What is the best environment to interview this pt in?
reassure safety, provide environment of safety
private area with door open/partially open

What is the therapeutic range for lithium?
0.6-1.2 mEq/L

Many postpartum women have SI after delivery, yet they don’t have psych evals before leaving the hospital, what could a possible solution be?
collaborating care between psych and OBGYN to provide the best care for that pt

If collaborating with a provider who is not in the same clinic or hospital, the informed consent [does/does not] apply?
Does not, ie pt will need to sign consent for labs/ chart to be sent

When discharging from inpatient you want [to fax/print off-give to pt.] labs for the outpatient provider before pt discharge?
fax; so the new provider has them for follow-up appointment

Before referring a pt out or discharging them, it is good to ask yourself what question?
is there something that I could do for this patient first?

What 3 things are MUST KNOWS about Lithium?
has neuroprotective effect for bipolar disorder

the gold standard for treating manic episodes

the mood stabilizer w/ anti-suicidal effects

What mood stabilizer has anti-suicidal effects?
Lithium

What 5 labs should be drawn with lithium?
Thyroid panel (TSH)
Serum creatinine
BUN
Urinalysis
HCG

What is a normal SerumCr range?
0.6-1.2 mg/dL

What is the normal range for BUN?
10-20 mg/dL

When should you suspect kidney disease with a urinalysis?
4+ protein; if pt on Lithium–>monitor closely toxicity

What are 7 side effects of lithium?

Hypothyroidism
Fine hand tremors
Maculopapular rash
GI upset (N/V/D, cramps, anorexia)
Polyuria, polydipsia, diabetes insipidus
T-wave inversions
Leucocytosis

What are 7 signs of lithium toxicity?

Severe nausea, vomiting, diarrhea

Confusion

Drowsiness

Muscle weakness

Heart palpitations

Coarse hand tremor

Unsteadiness while standing or walking

Bold =must knows

Which 3 drugs can increase lithium levels by decreasing renal clearance?
NSAIDs (ibuprofen, indomethacin)
Thiazides
ACE-I

What is a toxic level of lithium?
1.5 mEq/L or higher

At what level do you monitor lithium closely
(but not discontinue)?
1.3 -1.4 mEq/L or higher

With suspected lithium toxicity, what do you do first?

D/C Li and draw Lithium level (not VS)

What is the appropriate action if lithium level is 1.5 mEq/L?
D/C lithium
Check lithium level

What 2 conditions can increase lithium levels?
If this pt is going on a hike, what would you recommend ?
Dehydration
Hyponatremia

Take extra water to avoid dehydration

What birth defect can lithium cause?
Ebstein anomaly (congenital heart defect)

When working with adolescents, parents often feel they have a right to know what is going on with their child, but the teen has a right to confidentiality. This can create an
Ethical dilemma

When building a therapeutic alliance w/ adolescents, it is important to stress
confidentiality;
what they say remains confidential unless they are wanting to harm self/ someone else,

[or they are in a relationship with someone much older (like 14 y/o seeing a 30 y/o)]

if an adolescent C/O abuse, even if they make the claim in the presence of the parents, what should you do?
interview the teen w/o the parents; call CPS

What 3 neurotransmitters are associated with ADHD?
DA, NE, 5HT aka serotonin –>(DNS)

The Frontal cortex, Basal ganglia, Reticular Activating System are associated with which psychiatric disorder?
ADHD

Inattentive Type ADHD demonstrates abnormalities in which part of the brain?
Prefrontal cortex

What should be done before placing a patient on stimulants for ADHD (priority)?
Cardiac history

Excessive worry, apprehension, or anxiety about events or activities that occurs more days than not for a period of at least 6 months is
GAD

if ADHD and tic both present, what is contraindicated?
Stimulants

if a stimulant was tried and provided some but not enough benefit (helped, but not long enough) what can this mean?
the medication has been cleared from the body

What are the lower age limits for stimulants for ADHD?
AMPH = 3yrs MPH = 6yrs

When considering Clonidine or Guanfacine for a pt, the PMHNP knows?
Clonidine is more sedating than Guanfacine;
Guanfacine lasts longer than Clonidine

What does thought process assess?
thoughts and ideas

A normal thought process is described as
Logical
Linear
Coherent
Goal-oriented

Moving from thought to thought and never getting to the point is called
Tangentiality (“goes on tangent”)

Providing unnecessary detail but eventually getting to the point is called
Circumstantiality (“goes in circles”)

Themes that occupy a patient’s thoughts and perceptual disturbances is called
Thought content

Thought content includes 3 items
Suicidal ideation
Homicidal ideation
Hallucinations/Delusions

Asking a patient to count backward from 100 by seven (serial 7s), or anything going backwards measures what?
Concentration
Attention

Asking the year, season, date, month, and location measures
Orientation

Asking a patient to repeat “bed, red, ball” measures
Registration (ability to learn new material)

Asking a patient to repeat objects 5 minutes later measures
Recall (memory)

Asking the patient who is the president of the US or governor of state is measuring
Fund of Knowledge

What is a simple test that can be administered in a minute or two to measure possible dementia?
Clock Drawing Test

Impairments in the clock drawing test (CDT) can be associated with damage to
Right parietal lobe (right hemisphere)

Hyperactivity of dopamine in the mesolimbic pathway to the dorsolateral prefrontal cortex (DLPFC) modulates
positive psychotic symptoms

The safest drug switch strategy is to have a “washout period” of _ half-lives between cessation of old med and the introduction of the new med.

5 1/2

Decreased dopamine in the mesocortical pathway (dorsolateral prefrontal cortex) leads to what 2 symptoms?
Negative symptoms: affect, anhedonia, asociality, alogia, apathy, avolition,
Depressive symptoms

The nigrostriatal pathway modulates
motor movements

Dopamine has what type of relationship with acetycholine
Inverse

Dopamine blockage in the nigrostriatal pathway leads to what 4 side effects?
Acute dystonia: stiff neck, muscle spasms neck/back, painful
Parkinsonism syndromes
Akathisia
Tardive dyskinesia

Blocking DA in the tuberoinfundibular pathway leads to
Increased prolactin

What are 5 symptoms of hyperprolactinemia?
Amenorrhea
Galactorrhea
Sexual dysfunction
Gynecomastia
Osteoporosis (Long-term)

What antipsychotic has the greatest effect on prolactin?
Risperidone

What is the black box warning for carbamazepine?
Agranulocytosis and

SJS in Asians (+) for HLA-B*1502 allele

What are 8 symptoms of aplastic anemia?
Pallor
Fatigue
Headache
Fever
Nosebleeds
Bleeding gums
Skin rash
SOB

In which ethnic group must you screen for the HLA-B1502 allele before initiating CBZ (Tegretol) therapy? Why is it necessary? Asians risk for SJS in Asians (+) for HLA-B1502 allele

Which 2 drugs have the highest likelihood of causing agranulocytosis?
Clozaril
Carbamazepine

A sudden fever, chills, a sore throat, and weakness are symptoms of
Agranulocytosis

At what ANC should clozapine be stopped (even if asymptomatic)?
<1000

What is neutropenia?
<2000 PMNs or WBC <2000

What is agranulocytosis?
<500 ANC

What does DIGFAST stand for?
Distractibility

Indiscretion

Grandiosity

FOI

Activity increased

Sleep decreased

Talkativeness

What neurological symptoms might one see in Conversion Disorder?
Blindness
Mutism
Paralysis
Paresthesia (glove stocking syndrome)
Seizures

When do symptoms typically begin with Conversion Disorder?
After a stressful experience (suddenly)

What are 5 stressful events in a child’s life that can cause adjustment disorder?
Family move
Parent divorce/separation
Loss of pet
Birth of sibling
Sudden or chronic illness

A child with an Adjustment Disorder with disturbances of conduct may have what symptoms?
Not going to school, destroying property, driving recklessly, or fighting

What are some key sx of Oppositional Defiant Disorder (ODD)?
NOT aggressive (key difference vs CD)
loses temper easily
annoys others
angry/resentful
argues with authority
easily annoyed
blames others
spiteful
refuses to comply with rules/requests from authority figures

What is the mainstay treatment of Oppositional Defiant Disorder (ODD)?
Therapy, individual and family

What is the primary therapy used to treat Oppositional Defiant Disorder (ODD)?
Child and parent problem-solving skills training, boundary setting

A repetitive and persistent pattern of behavior in which the rights of others or societal norms or rules are violated is what disorder if under age 18?
Conduct Disorder

What are 3 “hallmark” characteristics of Conduct Disorder?
aggression

Destruction of property

Lack of remorse

What 2 factors differentiate Conduct Disorder from ODD?
Severity

Aggression

What are 4 pharmacologic treatments for Conduct Disorder?
Things to target mood and aggression

–Antipsychotics

–Mood stabilizers

–SSRIs

–Alpha agonists (Clon. and Guan)

What type of therapy is used in the treatment of Conduct Disorder?
Behavioral therapy/ problem solving skills

What is the goal of multisystemic family therapy (MFT)?
Reduce barriers to resources for youth with problematic behavior

What is Multisystemic family therapy?
Home-based model for youth (12-17) with serious antisocial problematic bx and criminal offenses by empowering parents with resources and skills and reducing barriers to resources that prevent families from accessing services needed for effective management of youth

helps to develop natural support systems

What is the range for MMSE?
score 0-30. Higher the better

25+ normal

21-24 mild

10-20 moderate

0-9 severe

What is the range for SLUM?
score 0-30 Higher the better

27+ normal

21-26 mild

<21 dementia

What is the range for HAM-D?
range 0-76

0-7 normal

8 mild

14 moderate

19 mod-severe

23+ severe

Just moderate 14-18

What is the moderate range for PHQ-9
range 0-27

0-4 normal

5 mild

10 moderate

15 mod-severe

20 severe

Just moderate 10-14

What is the moderate range for BDI?
range 0-63

0-9 normal

10 mild

19 moderate

30 severe

Just moderate 19-29

What is the moderate range for HAM-A?
0-56

0-17 mild

18 moderate

25 severe

Just moderate 18-24

What is the moderate range for GAD-7
range 0-23

0-4 normal

5 mild

10 noderate

15 severe

Just moderate 10-14

What is the moderate range for COWS?
13-24

If a patient’s COWS score is between 5 – 12, what do you do?
Administer clonidine (mild score)

If a patient’s COWS score is between 13 – 24, what do you do?
Administer buprenorphine (Suboxone) (moderate score)

When do you administer clonidine on the COWS?
Definitely at score of >7; prior to that (scores 5-6) it can be given

When do you administer buprenorphine (Suboxone) on the COWS?
moderate symptoms (13-24)

What is a moderate range on the CIWA?
16-20

When do you administer prn meds such as benzodiazepine or for N/V/D on the CIWA?
Mild symptoms (8 or greater)

When do you administer scheduled benzodiazepine +PRN’s on the CIWA?
moderate symptoms (15 or higher)

If a patient has severe depression (over 18 on HAM-D) (over 14 on PHQ-9) (over 29 on BDI), how do you treat it?

Medication and/or therapy

Assess for suicidal ideation

If a patient has moderate depression (14 – 18 on HAM-D) (10 – 14 on PHQ-9) (19 – 29 on BDI), how do you treat it?

Medication and/or therapy

If a patient has mild depression (<14 on HAM-D) (<10 on PHQ-9) (<19 on BDI), how do you treat it?

Therapy or nothing

If a patient has mild anxiety (<18 on HAM-A) (<10 on GAD-7), how do you treat?

Therapy or nothing

If a patient has severe anxiety (>24 on HAM-A) (>14 on GAD-7) how do you treat?

Medication and/or therapy

If a patient has moderate anxiety (18-24 on HAM-A) (10 – 14 on GAD-7) how do you treat it?

Medication and/or therapy

What are 4 areas in the brain that can cause aggression, impulsivity, and difficulty with abstract thinking?
Prefrontal cortex
Amygdala
Basal ganglia
Hippocampus

Abnormalities in the Prefrontal cortex, Amygdala, Basal ganglia, and
Hippocampus can cause what 3 symptoms?
Aggression
Impulsivity
Difficulty with abstract thinking

What are 6 symptoms of NMS?
*Extreme muscular rigidity

*Mutism

*Elevated labs CPK (creatine phosphokinase), LFT’s and WBC’s

Myoglobinuria

autonomic instability vBP, ^HR/RR

Fever

*= top 3 ways to differ from SS

Elevated CPK (creatine phosphokinase) in NMS is caused from
muscle contraction and destruction

What are 2 differentiating symptoms (Key indicators) of Serotonin Syndrome?
Hyperreflexia
Myoclonic jerks

S/S of serotonin syndrome include:

what are the 4 most often seen (in bold)?

“Shits and Shivers”

§ Diarrhea (shits)

§ Shivering,

§ Hyperreflexia/myoclonic jerks

§ Increased temperature

§ Vital sign instability

§ Encephalopathy

§ Restlessness/anxiety

§ Sweating

How do you treat NMS?
DC offending agent
Bromocriptine or
Dantrolene

What does Dantrolene treat in NMS?
Muscle rigidity

How do you treat Serotonin Syndrome?
DC offending agent

Cyproheptadine: H1 antihistamine that acts to block

5-HT1A and 5-HT2A receptors

What combination of medications increases the risk of Serotonin Syndrome? What herbal supplement also increases the risk?
What class of migraine medications raises the risk?
multiple antidepressants (SSRI/SNRI/TCA/MOAI)
St John’s wort –> with any of the above AD classes
Triptans –> with any of the above AD classes

When switching from an MAOI to SSRI/SNRI, how long should you wait? Why?
2 weeks;

for the MAO to regenerate (remember MAO-I deplete MAO)

When switching from Prozac to any antidepressant (TCA, SNRA, or MAOI), how long should you wait?
5-6 weeks

When switching from an SSRI/SNRI to a MAOI, how long should you wait? Why?
2 weeks; for the SSRI to degenerate

What is the function of compulsions in OCD?
Reduce subjective anxiety level

What is PANDAS? When would you suspect PANDAS?
Pediatric Autoimmune Neuropsychiatric Disorder associated with streptococcal infections in children

new onset OCD sx- recent strep throat

If a child has both intrusive thoughts and tics, his most likely diagnosis is
OCD

If a child has multiple tics (at least 2 motor and 1 vocal) for at least 1 yr, and no ruminating thoughts, his diagnosis is most likely?
Tourette’s, NOTE: (the tics do not necessarily have to occur at same time)

What 2 neurotransmitters are associated with OCD?
Serotonin
Norepinephrine

vocal tics
can include coughing, grunting, throat clearing, sniffling, or making sudden, vocal outbursts

A child between 7-17 years with a chronic dysregulated mood (moody for no reason), frequent intense temper outbursts, severe irritability, and anger is most likely to have what diagnosis?
DMDD

What are some symptoms associated with lead poisoning?
Developmental delay (learning difficulties)

Pica (paint chips)

irritability

gastrointestinal sx

low weight

Which 2 demographic characteristics are associated with lead poisoning?
rural areas
houses built in 1970’s (lead was in the paint)

A child with developmental delay who eats things (such as paint chips) may have
Lead poisoning

What 4 assessments should be done in a patient receiving an antipsychotic that causes weight gain?
BMI
Hip-to-waist ratio (waist circumference)
Glucose
Lipid panel

Where is norepinephrine produced?
Locus coeruleus
Medullary reticular formation

Serotonin is produced in
raphe nuclei in the brainstem

What 3 areas of the brain is dopamine produced?
Substantia nigra
Ventral tegmental area (VTA)
Nucleus Accumbens

Where is acetylcholine synthesized?
Basal nucleus of Meynert

What is the function of the amygdala?
Emotional memories (aggression, fear, anxiety, rage [think amygdala], stress)

What are 5 functions of the hippocampus?
Emotions

Stress

Learning

Motivation

Memory conversion ST to LT

BOLD =HY

What disorder is associated with persistent deficits in social communication and social interactions across multiple settings?
Autism Spectrum Disorder

A child who likes to line up, stack, or organize objects and toys in tidy rows, little or no eye contact and does not respond when called by name may have
Autism

What theory claims that dysfunction of a particular neuron system may be a cause of the poor social interaction and cognition in autism?
Broken Mirror Theory

What is the age criteria for a DSM-5 diagnosis of Tourette’s Disorder?
tics appeared before age 18 yrs

What is the pharmacological treatment of Tourette’s Disorder?
Antipsychotic
(haloperidol, pimozide, aripiprazole*)

What are the 3 primary neurotransmitters involved in Tourette’s Disorder? What is special about DA?
DA, NE, 5HT or (DNS)
hyperactivity of DA can lead to tourettes

What are 11 risk factors for suicide?
Previous suicide attempt

45 and older ( > 55 for women)

Male gender

Divorced, single, or separated

White (Caucasian)

Living alone

Psychiatric disorder

Physical illness

Substance abuse

Family history of suicide

Recent loss

bold (must knows)

What are 5 physical characteristics of Anorexia Nervosa?
Low BMI (below 18.5 is underweight)
Amenorrhea
Emaciation (abnormally thin)
Bradycardia
Hypotension

What is the BMI in a patient with Bulimia Nervosa
Normal range (18.5-24.9)

Intense anxiety and fear, helplessness, reexperiencing the event and avoidance behaviors within 4 weeks of a traumatic event is DSM-5 criteria for what disorder?
Acute Stress Disorder

What are the 3 hallmark symptoms of PTSD?
Intrusive re-experiencing of the trauma
Increased arousal (hyperarousal)
Avoidance of stimuli associated with trauma

What are 3 pharmacological treatments for PTSD
SSRIs
TCAs
Prazosin for nightmares

What 2 nonpharmacologic treatments for PTSD?
EMDR

CBT

What are the 12 components/symptoms of the COWS?
Pulse
Sweating
Restlessness
Pupil size
Bone aches
Running nose or tearing
GI upset
Yawning
Tremors
Anxiety/irritability
Gooseflesh skin

What are the 9 components/symptoms of the CIWA Scale?
Nausea/vomiting
Tremor
Sweats
Anxiety
Agitation
Tactile disturbances
Auditory disturbances
Headaches
Orientation

What are 3 treatments for Alcohol Use Disorder?
Acamprosate (Campral)
Disulfiram (Antabuse)
Naltrexone (Vivitrol, ReVia)

Which agent for the treatment of Alcohol Use Disorder is not metabolized by the liver?
Acamprosate (Campral)

What to avoid while taking Acamprosate and for at least 2 weeks post use?
anything with ETOH, mouthwash, aftershave, vinegars, perfume, cough/cold meds

What are 4 therapeutic factors in Yalom’s Group Therapy?
Instillation of hope
Universality
Group cohesiveness
Altruism

What is an approach to organizational change which focuses on strengths rather than weaknesses?
Appreciative Inquiry

What are the 3 goals of Quality Improvement?
Projects designed to
–improve systems,
–decrease cost,
–improve productivity

The process/strategy of Quality Improvement is called
PDSA Cycle

Plan

Do

Study

Act

Process, Policy reform, Policy environment, and Policy makers are the 4 components of
Health Policy

Changes in programs and practices in Health Policy is called
Policy Reform

What is the “policy environment” component of Health Policy?
The arena the process takes place in (government, media, public)

What is the “policy makers” component of Health Policy?
Key players and stake holders

What is the first action when developing Health Policy?

Assess/address organizational barriers and facilitators

A treatment approach that does not focus on full symptom resolution but emphasizes resilience and control over problems and life is called

Recovery Model

What are 3 characteristics of the Recovery Model?
Nonlinear recovery, continual growth and occasional setbacks; Learning from experience (e.g. relapse)

Self-directed -pt is not told what to do

Individualized and Person-Centered-pt is the center of the therapy

What is the SBIRT, and what is it used for?
Screening

Brief

Intervention

Referral

Treatment

Screens for Substance Use Disorders

What is the Tarasoff Principle
Duty to warn victim of imminent danger of homicidal patients…varies by state

What is the Rennie vs Klein Court case?

“Rennie’s Right to Refuse and appeal”

An involuntarily committed patient who has not been found incompetent, absent an emergency, has a qualified right to refuse psychotropic medications

What is the Donaldson vs. O’Connor court case?

“You can’t confine Donald”

You cannot confine (commit a person involuntarily) who is not imminently dangerous to self or others

What are 4 key components of Strategic Therapy?
Problem and Symptom focused

Paradoxical directive/intervention (reverse psychology) when pt non-compliant

Straight forward directive- when pt compliant

Reframe belief system

Miracle Questions, Exception-finding questions, and Scaling Questions are used in which therapy?
Solution-Focused therapy

Which therapy uses triangles/triangulation, and self-differentiation?
Family Systems Therapy

The paradoxical directive, a technique to be used with caution, is used in which therapy?
Strategic Therapy

Hierarchies, Boundaries, and Genograms are characteristics of what therapy?
Structural Family Therapy

Genograms are used in which 2 family therapies?
Family system therapy
Structural therapy

What type of therapy should be performed when a patient presents with a cultural syndrome?
Brief supportive therapy

What type of therapy should be used in a patient who has just lost their job?
Brief supportive therapy

What is the most important concept in working with patients from different cultues?
Respect

What should you teach a patient interested in meditation?
muscle relaxation

Which ethnic group views mental illness as an imbalance between an individuals’ relationship with world?
Native American

Which ethnic group has the highest incidence of suicidal attempt and completion?

Native American

A culturally expected response to a stressor is called
Cultural Syndrome

How does the PMHNP perform health promotion education in a community setting with different ethnicities
provide multi-cultural teaching/education by using ethno-specific assessment parameters

If a patient is regularly taking Kava, what lab should you get?
LFTs

Releasing information to a traditional healer
is okay after you get clearance for informed consent

Which antipsychotic has the least weight gain?
Ziprasidone (Geodon)

If your patient doesn’t understand English, the PMHNP should
get a translator/interpreter for them so they can understand you and know you are culturally competent

NativeAmericans have healing sticks that can heal them of illnesses, if the patient is hospitalized inpatient, what should you do?
inform/educate the staff members of cultural competency/sensitivity re: the healing stick, the healing stick doesn’t need to be taken away (make accommodations for the patient )

Which SGA is the least sedating?
Aripiprazole (Abilify)

Which 3 antipsychotic causes weight gain?
Quetiapine (Seroquel)

Olanzapine (Zyprexa)

Clozapine (Clozaril)

What is the first action to take with a patient who is gaining weight from an SGA?
nonpharmacologic

nutritional counseling (diet)

exercise

CBT

When should clozapine be discontinued?
ANC < 1,000 (even if asymptomatic)

One of your patients receiving psychiatric medication develops an infection. What do you consider?
Agranulocytosis

What antipsychotic is anti-suicidal in schizophrenia?

Clozapine

When a young woman is suspected of a toxic dose of lithium, what lab should you obtain initially?
HCG

If a patient has signs/symptoms of lithium toxicity but you have no labs available, what do you do?
DC lithium

A patient with what medical condition is at risk for lithium toxicity?
Cardiac failure (hyponatremia)

What are the key indicators for NMS?
Extreme muscle rigidity

Increased CPK

If a depressed patient is taking a triptan for migraines, what should you prescribe for depression?
Bupropion

What is first line treatment for MDD? Why?
SSRIs
Safer in OD

What are the 2 antidepressants of choice in a depressed patient with cancer? Why?
Citalopram (Celexa)
Escitalopram (Lexapro)
less risk for Drug-Drug Interaction

What is the antidepressant of choice in a depressed patient with sexual dysfunction?
Bupropion

What is the antidepressant of choice in a depressed patient with decreased energy?
Bupropion

What are 3 treatments for depressed patients with neuropathic pain?
SNRI

TCA (think safety tho)

Gabapentin/Pregabalin (alpha2 delta ligands)

BOLD = know

Which SSRI is most likely to cause insomnia?
Fluoxetine (Prozac)

What question is important to ask in the social history with a depressed patient?
Alcohol intake (self-medicating)

Suicidal ideation for children, adolescents, and young adults <24 years is a BBW with what medications?
all Antidepressants

What question should ask all adolescent patients at each visit?
Self harm (frequency and severity)

Which 2 groups is it especially important to ask about SI at each visit?
Adolescence
Schizophrenics

Alpha 2 adrenergic receptor blockers (Clonidine and Guanfacine) have lower tolerability in patients with schizophrenia. What effects from these meds are limited in this population?
the neuroprotective effects

Why do children have a decreased placebo response with antidepressants?
Few evidence-based studies

What is the onset of schizophrenia in males?
18-25

What is the onset of schizophrenia in females?
25-35

What question should ask schizophrenic patients at each visit?
SI

What is the most likely diagnosis in a patient with homicidal ideation?
Antisocial Personality Disorder

What would yo expect to see on an MRI/Pet scan in a patient with schizophrenia?
All structures decreased EXCEPT Ventricles, they are enlarged

Which class of agents should be avoided in patients with schizophrenia?
Stimulants (will increase DA)

Abnormalities, changes/deficits in which structures of the brain are associated with aggressive and impulsive behavior ?
Prefrontal cortex
Amygdala
Hippocampus
Basal ganglia
limbic Regions

What is Assertive Community Treatment (ACT)?
Form of rehabilitation post hospitalization for Severe Mental Illness (SMI)
Case management approach
24/7 services

Can patients receive Assertive Community Treatment (ACT) while in the hospital?

No; think “community” in ACT

What is the best treatment program for SMI patients with a long Hx of poor medication adherence?
Assertive Community Treatment (ACT)

Social skills training in a schizophrenic patient is what type of prevention?
Tertiary

What are 3 benefits of aerobic exercise in schizophrenic patients?
Increased cognition

mproves Quality of Life

improves Long-term health

What is a delusion?
A firm belief despite contrary evidence
Ex: Church members are part of a cult

How often is Haldol Decanoate typically adminitered?
monthly

If a patient is receiving Haldol 5 mg PO bid, how much Haldol Decanoate should they receive for the 1st month?
20 x [total daily dose] = first month dose
What is their maintenance dose?
20 x 10 mg = 200 mg (first month)
Maintenance: 10-15 x previous daily oral dose

When converting oral Haldol to depot form, what the maximum dose should administer at a time?
100 mg R/T adverse SE;
then 5-7 days later then give an additional 100 mg

How do you conduct a MSE in a preschooler (3-5 yo)
Clinical observation (listen/observe)

What is included in thought content?
SI/HI, plan

What are the 5 components of the MMSE (Folstein’s)
Concentration/attention/calculation
Orientation
Registration/ability to learn new material
Recall/memory
Fund of Knowledge

What pharmacologic characteristic makes SGA’s unique? What does this help prevent?
5-HT2A receptor antagonism
EPS

What antipsychotic should you give to a patient with their first psychotic episode?
If they are a harm to themselves or others how should it be administered?
What is the risk of this type of administration?
SGA

IM Invega, Geodon or Abilify

increase risk for EPS (acute dystonia)

Which 4 SGAs are available in IM form?
Ziprasidone (Geodon)

Olanzapine (Zyprexa)

Aripiprazole (Abilify)

Paliperidone (Invega)

What agent other than antipsychotics can increase prolactin and can cause TD?

Metoclopramide (Reglan)

What do you do if a patient is on olanzapine (Zyprexa) and stops smoking?
Decrease olanzapine dose (he stopped the inducer)

What effect does tobacco have on CYP450?
1A2 inducer

What class of drugs are primarily inhibitors?
Antibiotics

How does erythromycin and clarithromycin affect CYP450?
Inhibitors

What do you do if a patient on carbamazepine is given an inhibitor like erythromycin?
Decrease CBZ dose
(avoid aplastic anemia)

If a depressed patient is receiving a drug that is associated with depression (like interferon), what do you do?
Increase dose of antidepressant

What 7 signs/symptoms are associated with stimulant abuse?
Insomnia
Tremors
Irritability,
Mood swings.
Agitation,
Anxiety
CV (increased HR/BP, palpitations)

A patient who c/o pain after eating, bloating and fullness, what do you suspect?
Delayed gastric emptying from anorexia

What 2 classes of medications delay gastric emptying?
H2 antagonists: Famotidine (Pepcid)

PPIs: Omeprazole (Prilosec)

What 2 classes of medications interfere with the absorption of psychiatric medications and should be spaced apart by 2 hours?
Antacids
PPIs: Omeprazole (Prilosec), Pantoprazole (Protonix)

What is a common side effect of SSRIs in the older population
Anxiety

What is a paradoxical effect of benzodiazepines?
Increased anxiety
(avoid in future)

What is apotosis?
Cell death/neuronal loss

What is the greatest risk factor for bipolar disorder?
What is the first question you ask with a patient with suspected bipolar disorder?
genetics (very heritable)

fm Hx

What should you consider in a 45 y/o patient that presents with manic symptoms?
Medical condition

if a pt w/ SCZ is controlled on Risperdal; but has been prescribed a steroid by the PCP, what might you need to do?
increase the anti-psychotic

What are 2 differences between mania and hypomania?
More severe
Increased duration (7 days vs. 4 days)

A patient who is manic, irritable mood, and uncooperative is most likely to have what disorder?
bipolar disorder

What are key symptoms to consider in a patient you suspect of mania or hypomania?
Grandiosity or exaggerated self esteem
lack of sleep (and not tired)
talkativeness or pressured speech

What is a hallmark feature of borderline personality disorder?
Self-harming behavior (recurrent)

What therapy in borderline personality disorder been shown to decrease suicidal ideation?
DBT

Who developed DBT?
Marsha Linehan

In DBT, diagnosis of borderline disorder requires activity?
journaling/diary

A patient with borderline personality disorder with symptoms of depressed mood, rejection sensitivity, and emotional lability should be prescribed?

A patient with borderline personality disorder with symptoms of irritability, anger, self-harm could be given?
Valproic acid/Depakote/DVP

Lithium

A hospitalized patient with a reaction to a stressful event is most likely to have what diagnosis?
Adjustment Disorder

A child with a history of juvenile detention (e.g., fire setting, forced sex) is most likely to have what diagnosis?
Conduct Disorder

Is a child with ODD or Conduct Disorder more likely to need medication?
Conduct Disorder

If a child develops tics after beginning a stimulant, should he be diagnosed with Tourette’s?
No
(tics caused by a substance is an exclusionary diagnosis)

Are motor tics permanent?
Not always

What is factitious disorder imposed on another?
Munchausen syndrome by proxy, caretakers make up or produce physical illnesses in the individual receiving care. Usually parent –> child

What is required if you suspect a factitious disorder imposed on another (Munchausen syndrome)?
Report to CPS
Child abuse (duty to report)

What is Factitious Disorder
Faking illness to get medical attention or sympathy

A patient with factitious disorder is most likely to have what type of demographic?
Unmarried healthcare worker

What is malingering?
Feigning illness for secondary gain ($)

(not a psychiatric diagnosis)

A patient who claims disability or worker’s comp when there is no injury is most likely
Malingering (not a psychiatric diagnosis)

A child who has been neglected, abuse, or comes from a foster home, is most likely to have what diagnosis?
Reactive Attachment Disorder

A child who shows no emotion or is withdrawn may have what disorder?
Reactive Attachment Disorder

A disorder that presents like PTSD, but symptoms last less than a month is most likely to have
Acute Stress Disorder

How does a panic attack differ from Panic Disorder?

acute (peaks w/in minutes), impending doom

How does Panic Disorder differ from a panic attack?

Chronic, recurrent, unexpected

How do you treat a patient with panic disorder who is taking albuterol for asthma?

SSRI or benzodiazepine

(don’t give beta blocker)

What is the treatment of choice for acute symptoms of panic disorder?

Beta blocker (not addictive)

How do you treat test anxiety for maintenance therapy?
SSRI

How do you treat text anxiety acutely?
Beta blocker

Benzodiazepine

What structure of the brain is involved with attention, executive function, planning and cognitive processes (working memory, problem solving) that affects ADHD?
Dorsolateral prefrontal cortex

Children with inattentive ADHD are most likely to have an abnormality in which part of the cortex?
prefrontal

What do you recommend to a mother of a child with ADHD as an aftercare plan on admission?

Brief supportive therapy (not after dc)

If a patient present with irritability, depression, and lability, which questionnaire do you administer?
Mood Disorder Questionnaire (MDQ)

What disorder resembles bipolar disorder in children (7 and 17 years)?
Disruptive Mood Dysregulation Disorder (DMDD)

What should you assess initially in a child who presents with nightmares?

If other family members have the same problem

aftercare starts
upon admission

What are 2 key factors in the diagnosis of Autism Spectrum Disorder?
Persistent deficits in social communication and social interaction across multiple settings

A child that does not establish eye contact is likely to have
autism spectrum disorder

When is the only time you should give a benzodiazepine to a patient with delirium?
Alcohol or substance withdrawal

An older female has delirium and you suspect a UTI, what 2 labs should you order?
UA
C/S

An older patient who has had mental decline (>1 year) with chronic irritability and personality changes should be assessed for
Dementia

What are 2 labs that should be obtained in an older patient suspected of having a metabolic cause of dementia?
B12
Folic acid

What is the initial treatment for a patient with dementia and psychosis?
Nonpharmacologic treatment

If a patient with dementia does not respond to nonpharmacologic treatment, what do you do?
Administer SGA

What type of dementia is HIV-related? cortical or subcortical
Subcortical

A patient who presents with a cognitive deficit, decreased coordination, and behavioral symptoms should be assessed for what type of dementia?
HIV-related

In a patient with suspected HIV-related dementia, what is your initial action?
Obtain HIV test

What is first-line pharmacologic treatment for a patient with HIV-related dementia?
Antiretroviral treatment

An older adult who has had cognitive impairment for over a year and who tries to answer (may not be able to) or confabulates is most likely to have a diagnosis of
Dementia

An older adult who has had cognitive impairment for less than 5 months and who answers your questions with “I don’t know,” most likely has a diagnosis of
Depression (pseudodementia)

An older patient who has symptoms of irritability, agitation, hallucinations, and delusions is most likely to have
Depression (vs. dementia)

What are 2 ways to determine whether an older person with decreased memory has depression or dementia?
screening tools (SLUMS, Mini-Cog, MMSE)

onset of symptoms

A patient who presents with difficulties with executive function (poor judgment), slurred speech, personality and behavior changes, difficulties with language comprehension, lack of empathy, and poor social skills is most likely to have what type of dementia?
Frontotemporal (Pick’s)

A patient with visual hallucinations is most likely to have what type of demenetia?
Lewy Body

A patient who presents with cognitive dysfunction (decision making), lack of empathy, and difficulty with impulse control and emotions is most likely to have damage in what brain structure?
Anterior cingulate

A patient with damage to the cerebellum will demonstrate what?
Lack of balance

What condition may cause pica (eating things that are not food – paint chips)?
Lead poisoning

What 5 neurotransmitters are associated with mood disorders?
Dopamine
Norepinephrine
Serotonin
GABA
Glutamate

What do you do when an infant is about to die?
Give him to his parents to grieve

What 2 activities should be encouraged with grieving patients?
Family support
Support groups
(don’t give prescriptive advice)

Smoking, Caffeine, Low calcium/vitamin D in diet, Decreased weight,
and Lack of weight bearing exercises are risk factors for
Osteoporosis

long term prolactinemia is associated with?
Osteoporosis

Is Interpersonal Therapy offered individually as well as in group therapy?
Yes

If a patient is reluctant to participate in group therapy, what do you do?
Continue group therapy
Start individual therapy

Which is more effective – cognitive therapy or CBT?
CBT

Problem-solving, Skills training, Exposure therapy, Roleplaying/modeling, and relaxation are techniques used in
Behavioral Therapy

What are 2 common techniques used in CBT?
Cognitive restructuring
Journaling

Self-directed growth and self-actualization to find meaning in life are key concepts of what type of therapy?
Humanistic (person-centered)

What is the best therapy for dealing with conflicts with others (e.g., spouse, co-workers)?
Interpersonal therapy

How long does interpersonal therapy typically last?
12-16 weeks

If a patient is noncompliant, what is a technique that is used in Strategic Therapy?
Paradoxical directive/intervention

What is the benefit of paradoxical directive
Promote self-awareness (ie med adherence or fear of failure)

If a patient is compliant, what is a technique that is used in Strategic Therapy?
Straight Forward Directive

What type of therapy is a therapist using when he asks “what worked before when you had this problem”
Solution-focused therapy

How can a child’s body image, resilience, relationships, and social anxiety be improved?
Physical activity

When may a closed-ended question (yes/no) be necessary rather than an opened-ended question?
Children
(when a narrative cannot be constructed)

If a child does not respond or open or closed-ended questions, what to you do?
talk to parents

What therapeutic factor does open-ended questions instill?
empathy

Example of open ended questions
How did you fall?
Why do you think you are feeling sad?
Tell me more

If only the wife shows up for a couple appointment, what do you do?
Reschedule the appointment

A mother reports that her child was sodomized. What 2 things do you do?
Contact CPS
Crisis therapy

A mother reports her 5-y/o was raped by his 16 y/o brother. What 2 things do you do?
crisis therapy
separate brothers

What lab do you draw in a patient with rheumatoid arthritis
ESR

What does PICOT stand for?
P- Patient population of interest
I – Intervention of interest
C – Comparison of interest
O – Outcome
T – Time

How do you provide evidenced-based care?
Have access to current journals

A baby with a shrill cry is likely to have
Increased intracranial pressure

What do you tell a mother when she reports her preschool child is playing with their genitals?
That is normal between 3-6 yrs old (Phallic stage)

What do you tell an male adolescent who is concerned about nipple swelling and tenderness and breast enlargement?
This is normal between 9-16 yo.
It will disappear within 6 mo.

How do you treat decreased sexual drive in a older female patient?
Testosterone
(improves blood flow to pelvic region)

Why are women more likely to get intoxicated and have alcohol-induced hepatotoxicity?
Decreased alcohol dehydrogenase

What primitive reflex causes a newborn to grasp vigorously any object touching the palm or fingers or placed in the hand? when does it go away?
Grasp/palmar reflex; disappears by 5-6 mons

Infant reflex where a baby will startle in response to a loud sound or sudden movement is called______and it disappears by _?
Moro/startle reflex; disappears by 5-6 mons

What is the Babinski (Plantar) reflex? When does the Babinski (Plantar) reflex disappear?
the big toe is extended and the other toes fan in response to the brushing of the sole of the foot
disappears at 2 yrs

What is the absorption rate of PDE-5 inhibitors (Viagra)?
Rapid absorption

What 2 deficiencies are associated with
macrocytic anemia?
Folic acid
B12

With normocytic, macrocytic anemia, what 3 labs do you obtain?
B12, folic acid, Ferritin(iron)

What deficiency is associated with microcytic anemia?
Iron

Acute or chronic blood loss (GI bleeding, heaving menses) can cause what type of anemia?
Normocytic

When vital signs are not normal in a thin women, you suspect
Anorexia nervosa

You see a patient with suspected anorexia nervosa. What do you do?
Refer out for medical evaluation

If a patient with anorexia nervosa refuses to see a PCP, what do you do?
Contact CPS

If AN patient is medically unstable and parents refuse hospital treatment, what do you do?
contact CPS

What alternative medicine treatment is used with pain and depression?
Acupuncture

What act prevents a patient from unlawful hospitalization (can leave AMA)?
Habeus corpus
(usually max of 28 days)

A pedi patient presents with paresthesia, fatigue, unsteady gait, confusion, arm/leg weakness and asymmetric body movement of extremities. What do you suspect and what do you do?
Disseminated Encephalomyelitis
Neuro exam

If a patient is unable to remember events, how do you establish a diagnosis?
Provide anchor (memorable events) to establish diagnosis

How do you implement policy affecting NPs nationwide?
Host online forum/give survey to obtain information (vs. letters)

How do you implement policy that coworkers are against?
Tell them how the policy will increase quality of patient care

What is the initial step in providing continuous improvement of Quality of Care in an outpatient setting?
Create an instrument to monitor outcomes

What will you see in a laboratory test in a patient with an autoimmune disease?
Increased cytokine levels

What are 2 ways to treat enuresis (night-time bedwetting)?
Alarm clock
Desmopressin (DDAVP)

What is the mechanism of action of Desmopressin (DDAVP)?
Decreases urine production
(via antidiuretic hormone – vasopressin)

When considering the relationship between receiving samples from a drug rep and sponsored education, what is there a concern for, and what should you do?
Conflict of Interest
Create policy to address COI

A conflict of interest may occur in what 2 scenarios?
Receiving samples of drug rep
Attending an industry sponsored educational program

A patient calls because he read on the internet about a BBW on his medication. What do you do?
You don’t need to discontinue drug.
Research benefits and risks

What are 2 things you should do when you prescribe off-label?
Document
Provide full disclosure (risks/benefits)

When you prescribe trazodone to a male patient, what should you do?
Warn about priapism and document

Smoking, increased weight/obesity, and DM are risk factors for what disorder?
Sleep Apnea

What is tolerance?
Decreased effect with continuous use

What is the goal or Reflective Practice?
Improve practice

Providing debriefing techniques, feedback to staff and strategies to learn from experiences is called

Reflective Practice

If a patient has mild symptoms on the COWs, what should you do?
Give clonidine or a muscle relaxant

If a patient scores 8 or more on the COWS during opiate withdrawal, what should you do?
Administer clonidine or a muscle relaxant

If a patient has moderate symptoms on the COWS what should you do?
Administer buprenorphine/naloxone (Suboxone)

When should you administer buprenorphine/naloxone (Suboxone) in a patient withdrawing from opiates?
Moderate symptoms (13 or higher)

If a patient scores 13 or higher on the COWS scale during opiate withdrawal, what do you do?
Administer buprenorphine/naloxone (Suboxone)

Why should methadone not be used to treat opiate withdrawal?
Arrhythmias

Which benzodiazepine is used most often to treat alcohol withdrawal?
Diazepam (longer half-life)

If a patient with liver dysfunction or hepatitis is undergoing alcohol withdrawal, what benzodiazepine should he receive?
Lorazepam

If a patient presents with opiate withdrawal, what should you do?
Refer to residential center to administer protocol (don’t initiate in outpatient setting)

If a pregnant patient presents with opiate withdrawal, what do you do?
Refer to residential center (not CPS)

What do you suspect when a patient presents with CNS activation (hand tremor, insomnia, anxiety, psychomotor agitation, seizures) and

Autonomic hyperactivity (sweating, increased HR, increased BP) and

N/V and hallucinations/illusions?

alcohol withdrawal

What do you suspect when a patient who presents with muscle cramps, bony aches, diarrhea/abdominal cramps, rhinorrhea, lacrimation, goosebumps, pupil dilation increased BP/HR and yawning?
Opiate withdrawal

adolescents have a right to confidentiality, but not
privacy

Instillation of Hope, Universality, Altruism, Increased development of socialization skills, Imitative behaviors, Interpersonal learning, Group cohesiveness, Catharsis, Existential factors, and Corrective refocusing are

BOLD= focus on more

Therapeutic Factors which differentiate group therapy from individual therapy

Experiencing optimism through observing the improvement of others in the group is which of Yalom’s therapeutic factors?

Instillation of Hope

Learning to give to others is which of Yalom’s therapeutic factors?

Altruism

Sharing experiences/feelings in group clients realize they are not alone is which of Yalom’s therapeutic factors?

Universality

The “natural laboratory” is which of Yalom’s therapeutic factors?

Increased development of social skills

Group members modeling their behavior after other members of group/therapist is which of Yalom’s therapeutic factors?

Imitative behavior

Developing a sense of belonging in the group is which of Yalom’s therapeutic factors?

Group cohesiveness

Interacting with others to increase adaptive interpersonal relationships is which of Yalom’s therapeutic factors?

Interpersonal learning

Group members dealing with the meaning of their existence is which of Yalom’s therapeutic factors?

Existential factors

Participants reexperience family conflicts, allows them to recognize and change/correct behaviors that may be problematic is which of Yalom’s Therapeutic Factors?

Corrective refocusing

Openly expressing their feelings which were previously suppressed is which of Yalom’s Therapeutic Factors?

Catharsis

Focusing on the strength of the group is key to what organizational change strategy?

Appreciative Inquiry

Who determines the Scope of Practice?
State legislature (BON)
(varies state to state)

Who determines the Standard of Practice?

ANA
(doesn’t change)

What are 8 exceptions to confidentiality? (know all)
Unconscious (to save a life)

Intends to harm self/others

Court order (subpoena, summons)

Attorneys in litigation

Insurance companies

Mandatory reporting (diseases)

Tarasoff (duty to warn)

Child or elder abuse

What are the 5 elements of informed consent?
Nature and purpose of treatment/procedure
Risks and benefits of treatment
Risks and benefits of not undergoing treatment
Alternative procedures or treatments
Diagnosis and prognosis

What are the 7 ethical principles?
Justice
Beneficence
Nonmaleficence
Fidelity
Autonomy
Veracity
Respect

Fairness (according to gender, orientation, or ethnicity) is what ethical principle?
Justice

Discontinuing a medication that causes side effects is an example of what ethical principle?
Nonmaleficence

Denying a benzodiazepine to a patient with a SUD is an example of what ethical principle?
Nonmaleficence

“Doing good” and promoting well-being is what ethical principle?
Beneficence

Giving a patient a medication to treat symptoms is an example of
Beneficence

Being true and loyal is what ethical principle?
Fidelity

“Telling the truth” is what ethical principle?
Veracity

A patient’s right to refuse medication is an example of what ethical principle?
Autonomy

Treating everyone with equal respect is what ethical principle?
Respect

What are 2 important roles to perform as a client advocate?
support the client’s best interest while respecting family’s role
reduce stigma of Mental illness

What is the best way to reduce stigma?
Education (esp. large audience)

A work environment that improves patient safety through continuous learning designing safe systems is

Just Culture

What treatment approach does not focus on mental illness, is nonlinear (occasional setbacks), and stresses learning from experience?
Recovery Model

In the recovery model, the clinician focuses less on the Dx and tries to focus/foster the the pts __?
interests, abilities and dreams

What is the PDSA cycle
A quality improvement process or strategy

What is an example of a Quality Improvement Project?
RETROSPECTIVE Chart review

When implementing health policy, what is the first thing you do?
Assess barriers and facilitators
(prior to meeting with stakeholders)

What should you do before warning someone about harm (Tarisoff act)?
Contact Board of Nursing

A key component of Piaget’s Sensorimotor stage is
Object permanence

Object permanence develops in which of Piaget’s developmental stages?
Sensorimotor

What are 2 key components of Piaget’s Preoperational Stage?
Magical thinking
Symbols and language
thoughts of monsters being real, if they think of someone getting hurt and it happens, they think they caused it.

According to Piaget, at what stage do children display magical thinking and use language and symbols more?

Preoperational

What is a key component of Piaget’s Concrete Operations Stage?
See things from other’s perspectives (less egocentric)
develops concepts of
–conservation: clay is always clay
–reversibility: water to ice turns back to water

According to Piaget, during what stage do children see things from other’s perspectives – less egocentric?

Concrete operations

Conservation and reversibility occur during what stage per Piaget?
Concrete

What is the key component of Piaget’s Formal Operations stage?
Abstract logic (science projects, hypotheses)

According to Erikson, what period (age) is the development of meaningful relationships important?

Infancy (0 – 1 yr.)

According to Erikson, what period (age) is self-control a key component?

Early childhood (1 – 3 yrs.)

According to Erikson, what period (age) is being a “self-starter” (self-directed) a key factor?

Late childhood (3 – 6 yrs.)

According to Erikson, what period (age) is personal sense of identity a key factor?

Adolescence (12 – 20 yrs.)

According to Erikson, what period (age) is competence important, especially with classmates?

School age (6 – 12 yrs.) Industry vs inferiority

According to Erikson, what period (age) are committed relationships a key factor?

Early adulthood (20 – 35 yrs.) intimacy vs isolation

According to Erikson, what period (age) is the ability to care for others important (e.g., parenting)?

Middle adulthood (35 – 65 yrs.) generativity vs stagnation

According to Erikson, what period (age) does fulfillment and comfort with life occur?

Late adulthood (>65) Integrity vs despair

What 3 neurotransmitters are associated with MDD?
DNS (all decreased)

What 5 neurotransmitters are associated with bipolar disorder?
DNS
Glutamate (increased)
GABA

What 2 neurotransmitters are involved with Addictive Disorders?
DA (decreased)
GABA (decreased)

What 2 neurotransmitters are involved with OCD?
5HT (decreased)
NE (decreased)

What 4 neurotransmitters are associated with schizophrenia?
DA (increased)
Glutamate (increased)
GABA (decreased)
5HT (decreased)

What 3 neurotransmitters are associated with Autism Spectrum Disorder?
Glutamate (increased)
GABA (decreased)
5HT (decreased)

What neurotransmitters are associated with anxiety?
NE (increased)
5HT (decreased)
GABA (decreased)

What neurotransmitters are associated with Alzheimer’s?
ACh (decreased)
Glutamate (decreased)

What neurotransmitters are associated with Parkinson’s Disease?
DA (decreased)
ACh (Increased)

What 4 psychiatric disorders have dysregulation of dopamine, norepinephrine, and serotonin (DNS)?
ADHD
MDD
BPD
Tourette’s

Which 5 psychiatric disorders have dysregulation of the GABA receptor?
BPD
Schizophrenia
Autism
SUD
Anxiety

What 4 psychiatric disorders have dysregulation of glutamate receptors?
BPD
Schizophrenia
ASD
Alzheimer’s

Which 2 psychiatric disorder have dysregulation of acetylcholine receptor?
Alzheimer’s
Parkinson’s

What 3 disorders have dysregulation of glutamate and gabapentin?
autism spectrum disorder
bipolar disorder
schizophrenia

You are concerned a 5-yo child may be sexually abused. How do you conduct the interview?
Interview child and parents separately if child can communicate

You see a child playing with a toy in a sexual manner. What do you do?
contact CPS

What are the 2 most important aspects of interviewing an adolescent?
Rapport
Confidentiality (explain exceptions)

How should an adolescent be interviewed
separately form parents

If an adolescent is dating someone the same age, do you need to tell the parents?
No

If an adolescent is dating someone of the same gender, do you need to tell the parents?
No

What should you ensure before discharging a patient from the hospital?
can they stay safe and cope effectively

Should a patient be interviewed in a day room?
No (not confidential)

When do you draw a T3 or T4
If TSH is abnormal

What symptom occurs with high levels of ammonia?
confusion

What 4 symptoms are similar with serotonin syndrome and NMS?
Hyperthermia
Diaphoresis
Tachycardia
Altered LOC

Thoughts of self-harm are greatest in what 5 disorders (according to order) [I left this one…but I’ve not heard of it ranked]
MDD
BPD
Alcohol use disorder
Eating disorder
schizophrenia

Are oral contraceptives inducers or inhibitors?
inducers

A person with pressured speech, no sleep and inflated self-esteem, may suspect
bipolar disorder

If you can determine an identifying factor (an event) to why a person is depressed, what is their most likely diagnosis?
Adjustment disorder with depressed [or anxious, both etc.] mood

Can tics be a normal behavior?
yes

Language and memory are the 2 primary deficits in which type of dementia?
What are 2 examples?
Cortical
ALZ and Mad cow Ds

Motor and behavioral (depression, irritability, apathy) are the 2 primary deficits in which type of dementia?
What are 3 examples?
Subcortical
HIV dementia, Huntington’s and Parkinson’s

What is the value of integrative medicine (E.g., ObGyn + PMHNP)?
increased mental health access to those who need it most

What is the most important factor in a child who is grieving?
Intact family system

How do you treat a child who is grieving?
Normalize grief and loss
(don’t tell them what to do/how to grieve)
(responses vary)
supportive group therapy

How do you apply your knowledge of a relationship between ADHD and substance abuse to practice?
Screen ADHD patients for substance use
Screen SUD patients for ADHD

When you look up data about a patient in social media, what are you violating?
patient’s trust

Prior to sending prescriptions to a pharmacy, what do you need to do?
obtain patient consent

If patient is unable to give consent, what do you do?
assess for involuntary treatment (don’t forcefully give medication)

The ethical principle that is associated with preventing imminent danger to patient is?
nonmaleficence

Sending a patient to a crisis stabilization unit rather than a hospital is honoring what patient right?
least restrictive environment

You plan to teach the patient about their illness. What do do you do first?
assess knowledge

In what age groups should you obtain HCG?
12-51

What are normal prolactin levels for males and females?
Male: <20
Female: <25

serotonin is implicated in what 2 areas?
mood and sleep

What 2 symptoms might you see in a patient with a frontal lobe tumor?
social skill deficit
inappropriate affect

What is the term for symptoms caused by a medical treatment or drug therapy? How do you assess it?
Iatrogenic S/S, assess medical history to ID if S/S are caused by the medication or not

If you refer a patient to a residential center for alcohol withdrawal, what can you provide them for symptom control?
Hydroxyzine or buspirone (not benzodiazepine)

What does perjury mean?
Lying or withholding information under oath

Which 2 parties can you not disclose information without the patient’s consent?
family member
another provider

As part of reflective practice, what strategy is used post event?
debriefing strategies

When a medication error occurs, what is the first thing to assess per just culture?
Pt safety; assess the patient

What cardiac SE is geodon (ziprasidone) known for?
QTc prolongation

the max dose of Citalopram is 20mg/day in the elderly client, why?
risk for QTc prolongation (Note:max 40mg in adults <65)

Parkinson’s disease is caused by
damage or loss of the dopamine-producing cells of the midbrain, leading to dopamine depletion in the basal ganglia

Pseudoparkinsonism
caused by DA blockade in the nigrostriatal pathway causing sx resembling Parkinson’s Ds; tremor, shuffling gait, drooling, rigidity

In a trauma focused cognitive interview, what questions help build rapport?
non-judgmental questions
“how has that trauma affected you?”
“When did this happen”

There are 3 phases to EMDR, they are…
“DIB”
Desensitization
installation
Body Scan

the goal of EMDR is
to achieve adaptive resolution

Having the pt visualize the trauma, verbalize the negative thoughts or maladaptive beliefs; remain attentive to physical sensations while the also maintaining rhythmic eye movements is part of which EMDR Phase?
Desensitization phase

Instructing the pt to block out negative thoughts, to breathe deeply, and verbalize what they are thinking, feeling, or imagining occurs during which EMDR Phase?
Desensitization phase

Which EMDR phase closely resembles Cognitive Therapy and why?
Installation phase; because replacing negative thought with positive thought

Having the pt visualize the trauma alongside the positive thought, then scan their body for tension is which EMDR phase? Why is this important to do?
Body Scan; successful resolution will result in the pt having no body tension when seeing the trauma and the positive memory side by side; if (+) tension, pt is still working through to achieve resolution

telemedicine/telehealth legal issues include
confidentiality

med errors

Jurisdictions

Authority over licensure

if the PMHNP want to change an old Tx for a newer way to Tx a condition, what should it be based on?
the change should be evidence based

How are rating scales best used?
At baseline -pre-treatment, then at regular intervals to monitor response to Tx

When deciding if a client is ready to move to a less intensive level of treatment (Inpt to PHP or PHP to IOP etc); what does the PMHNP look for?
signs not ready: no coping skills, still blaming others

Signs they are ready: (+) coping skills, taking accountability

prevent/promotion, classes, safety initiatives, education, classes, modifying environment is what level of prevention?
Primary

screen-early detection, crisis hotlines, disaster is what level of prevention?
Secondary

treat- to prevent further deterioration, rehab, restoration, day treatment, social skills is what level of prevention?
Tertiary

When questions ask for a priority action…think about…
ABC, airway breathing, circulation
Maslows hierarchy

If undecided on an answer due to high similarities, choose:
the umbrella answer

What is the most common side effect of olanzapine/zyprexa
metabolic syndrome

what is the difference between typical and atypical antipsychotics
Atypical 5HT2A specific

1st psychotic break… two actions to take
UDS and r/o sub
Consider IM Geodon or Invega

Three AP with least weight gain
Latuda, Abilify, Geodon

Always encourage interprofessional collaboration
between therapists/pcps/SW/RN, the ENTIRE team

TSH High, then….t3/t4
T3, T4 low

TSH low, then…t3/t4
T3, T4 high

cold/hot sensitivity with t3/t4 relationship
T3/T4 low, hypothyroid, cold, slow
T3/T4 high, hyperthyroid, hot, flushed, tachy

What birth defect can be caused by depakote?
Spina bifida

What organ does depakote cause toxicity? and what sx would you expect to see? labs to run?
Hepatotoxicity: RUQ pain, reddish brown urine-
Do LFTs

kava kava is used to treat
anxiety and insomnia

Rash and fever associated with tegretol, suspect

What allele is HLAB 1502 associated?
Asians. They CANNOT have tegretol. Test all asians for this allele.

what rare and dangerous side effects are associated with tegretol
Aplastic anemia
Agranulocytosis-DC at ANC less than 1000

Sx’s of agranulocytosis
unusual bleeding or bruising, mouth sores, infections, fever, sore throat, fatigue

if starting a woman on lithium what test should be done? why?
HCG–risk of ebstein anomaly

adverse s/e of lamictal/lamotrigine
SJS

labs to checke BEFORE starting on lithium
BUN
CRE
urine protein

What does protein in urine indicate
kidney impairment; 4+ protein in urine=you cannot start on lithium

best choice med for decreasing si in bipolar disorder.
lithium

best choice med for si in schizophrenia
clozaril

best choice med for SI in borderline
lithium

What medications will INCREASE Li levels
NSAIDS
ACE’s
Thiazides/HCTZ

Besides medications, what else can cause increased Li levels
dehydration
hyponatremia
lithium s/e inc N/V, which will effect electrolytes, and dehydration status

what type of tremors will you see with lithium toxicity?
course tremors

lithium can cause what other comorbidities?
hypothyroidism
maculopapular rash
leukocytosis
twave inversion

what is a defining characteristic of NMS vs SS
muscle rigidity

Sx’s/labs associated with NMS
Inc CPK, WBC, LFT
Rhabdomyolosis
myoglobinuria
Can lead to mutism

myoglobinuria/rhabdo can cause cherry colored urine

Treatment for NMS and what each does
DC the offending agent
bromocriptin-D2 agonis
dantrolene: muscle relaxant
Make sure if ? is asking for agonist or relaxant

Sx of SS
HYPERREFLEXIA
myoclonic jerks

treatment for SS
ciproheptadine

how to best PREVENT SS
follow proper transition protocols
SSRI to MAOI=14 days
Prozac to MAOI=5-6 weeks
Triptans (Imitrex sumatriptan) can also cause SS due to serotonin increase with the use
give any NDRI welbutrin

Why are SSRIs considered the safest for use in depression
safest for OD first line treatment for depression

antidepressant for cancer
citalopram and escitalopramless drug to drug interactions

depressed patient presents with fatigue and low energy, consider:
NDRI wellbutrin

sexual s/e with ssri? try…
wellbutrin due to lower risk of sexual s/e

What medication must be avoided if client has seizure history or eating disorder? why?
wellbutrin due to decreasing the seizure threshold

if client has depression and neuropathic pain
SNRI or TCA for treatment of BOTH

What med class treats neuropathic pain well
alpha 2 delta ligands
Gabapentin
Lyrica

What medication class is good for depression with comorbid CA
SSRI
least chance of drug drug interactions
Celexa and lexapro are good choices

Black box warning on SSRI
increase SH kids, and young adult

Required education for rx ssri
long time for effect
side effects esp n/v/d
NO ABRUPT stopping d/t Serotonin discontinuation syndrome

what to ask when client is depressed (additional question)
alcohol use

Sx’s of serotonin discontinuation syndrome
fever, shivering, muscle aches and nausea diarrhea, agitation, cog impairment… (think flu like sx’s)
disequilibrium

What are some scenarios that place patients at risk of a hypertensive crisis?
MAOI and tyramine
MAOI and TCA
MAOI and Atypical AP
MAOI and decongestant
MAOI and stimulants
MAOI and asthma meds

Sx of Hypertensive crisis
HA
Diaphoresis
fever
facial flushing
pupillary dilation
palpitation

Treatment for HTN Crisis
DC Agent
Give Phenolamine

age of onset for male / female schizophrenia
male 18-25
female 25-35

schizophrenia has high rate of suicide
always be assessing for SI

what is the cause of schizophrenia
inadequate synapse formation
excessive pruning of synapses
intrauterine insult (drugs/toxin/viral agent/malnutrition/substance use, mental illness, o2 deprivation)

on MRI /PET schizonphrenia
ventrical enlargement

positive sx of schizophrenia caused by…
excess DA in mesolimbic pathway

positive sx of schizophrenia on a stimulant?
will potentiate DA release (worsening schizophrenias positive symtpoms) bc stimulants tap into the reward and addiction pathway inc dopamine

What is ACT
Assertive community treatment
Post hospital DC
not in the hospital

delusions, you respond how?
do not try to disprove
If HARMFUL delusions, notify authority
Also notify potential victim

how to assess mental abstraction
interpret a proverb

assess thought process why? and potential findings
to assess organization of patients thoughts
tangential: no rip to a ?
circumstantial: gets around to anss after going in circles with unnecessary details

MMSE thought content include
SI HI
plan
Hallucinations
delusions

MMSE/ Folstein Test assesses?
tool used to assess cognitive status in adults

concentration/attention/calculation assessed how?
spell a word backward or serial 7’s

Registration/ability to learn new material assessed how?
remember 3 words

orientation is assessed by
Person place time

fund of knowledge assessed how?
Who is the president? Governor?

clock drawing assesses?
takes 1-2 minutes
easy to administer
tests right hemisphere health,cannot do it they have a prob in R side.

First generation AP
haldol
fluphenazine
Chlorpromazine
thioridazine

2nd gen AP
rispeidone
olanzapine
seroquel
abilify
ziprasidone
lurasidone
clozipine

excess DA in mesolimbic pathway
positive sx of schizophrenia

decreased DA in mesocoritcal pathway
negative sx of schizophrenia
anhedonia
mask face
slow speech
isolative

nigrostriatal pathway
excess DA in this pathway but it doesn’t cause any sx,
but when DA is decreased in this pathway due to AP (dopamine blockade) increases acetylcholine=EPS sx

Tuberoinfundibular pathway
excess DA is normal here
but when it is decreased with AP leads to increased prolactin and hyperprolactinemia

what does high prolactin levels lead to
breast DC
amenorrhea
osteoporosis
breast enlargement

which AP is most closely associated with hyperprolactinemia
risperidone

Male prolactin
female prolactin—normals
male less than 20ng/ml
female less than 25ng/ml

EPS caused by
DA blockade in the NS pathway

acute dystonia
acute sustained contraction of muscles, usually of the head and neck
spasms
painful

treatment of acute dystonia
cogentin/benztropine IM, but may need Oral follow up for several days

akathisia
ANXIETY
restless
cannot sit still
pacing rocking

1st line treatment for akathisia? And which population to AVOID use in? 2nd line? 3rd line?
propranolol/or beta blocker
client with asthma on bronchodilators due to increased risk of bronchospasm when given beta blockers

2nd line: cogentin/benztropine

3rd line: benzos

akanesia
difficulty iniating motion

treatment for akanesia
cogentin/benztropin

Sx of akinesia
DIFFICULTY INITIATING MOVEMENT;
PSEUDO PARKINSONIAN SX’S:
muscle rigidity
shuffling gait
mask like facial expression
affect may be flat or blunted
pill rolling tremors (motor slowing)

Tardive dyskinesia (TD)/timeframes
chewing/lip smacking
facial dyskinesia
Treatment: dec dose of offending agent or switch to a different AP or can switch to clozaril.
can have onset between a few weeks and 2 years post starting of medication

Reglan (metoclopramide) and compazine (prochlorperazine) can both cause?
TD

what medication can worsen TD?
benztropine/cogentin

Inducers
cause low serum levels

inhibitors
cause high serum levles

reglan and compazine can both cause
TD

Smoking and dosing of meds
smoking is a strong inducer.
therefore smoking increases drug metablolism and you may need to dose higher. If they stop smoking serum levels will increase.
Always assess for smoking/cessation of.

Antibiotics/macroglides are INHIBITORS and greatly effect what medication
tegratol

what meds can cause mania
disulfram
steroids
isoniazide
antidepressants

what meds can cause depression
steroids
beta blockers
interferon
accutane-can cause birth defects too

if patient taking meds for a mood disorder and is on flonase or prednisone. What do you do with your dose of oxcarbazepine
increase the dose to adjust for the medication (steroids) influencing mania

treating a patient with zoloft and starts taking interferon. what do you do with zoloft dosing?
increase dose of zoloft to adjust for the increase in depression for the interferon

addiction neurotransmitters
DA
GABA

Gamma-aminobutyric acid main function
major inhibitory neurotransmitter

Sx’s of stimulant abuse
irritability
insomnia
tremors
delayed gastric emptying-feeling fullness/bloated

Pt with anorexia complains of pain after eating/bloating/fullness
Signs of delayed gastric emptying

what meds can delay gastric empying?
PROTON PUMP INHIBITORS
famotidine
omeprazole
ranitidine

ANTACIDS /PPI do what to psychotropic medications?
Decrease absorption of psychotropic medication

advise client to take other meds 2 hours AFTER antacids/PPI

Older adults and SSRIs for anxiety?
May increase anxiety
paradoxical effect

Older adults and benzos?
May increase agitation
paradoxical effect

apoptosis
neuronal loss or cell death

BP1 pneumonic
DIGFAST

DIGFAST
Distractible
Impulsive choices
Grandiosity
Flight of ideas
Active
Sleep not needed
Talkative

Borderline sx in regard to impulsivity
impulsivity is often with associated with recurrent SI and self harming behavior

Therapy for BPD/goal/founder
DBT
to decrease recurrent SI
Marsha Linehan

Conversion Disorder (Functional Neurological Symptom Disorder)
can be result of a stressful experience

present with neuro sx:
parasthesia
paralysis
blindness
Mutism

adjustment disorder (anxiety/depressive/mixed)
an emotional disturbance caused by ongoing stressors within the range of common experiences (dx of a new disease) (recent move) (loss of loved one)

factitious disorder
Condition in which a person acts as if he or she has a physical or mental illness when he or she is not really sick.
Ie drinking contaminated urine

Reactive Attachment Disorder
common in kids from foster care
kid goes back to real parent and may appear withdrawn, no emotions toward caregiver,
FOSTER is key word
Doesn’t seek comfort when distressed

ODD
NO AGGRESSION in ODD
defiant to authority
deliberately annoy others

treatment for ODD

  • Family Therapy
    with emphasis on child management skills

CD conduct disorder
no remorse
violent
raping/beating
theft
arson
INTENSE

AGE 6-17
When 18; think of Antisocial personality disorder

Treatment for CD
Medication and therapy
to target symptoms of mood and aggression

Alpha agonist aka alpha 2 adrenergic receptor blockers (clonidin, guanfacine)

Family therapy

acute stress disorder
An anxiety disorder in which fear and related symptoms are experienced soon after a traumatic event and last LESS THAN 1 MONTH

Over 1 month is PTSD

PTSD cluster of sx
Increased arousal
Reexperiencing the event
avoidance
nightmares

nightmare trmt in ptsd
prazosin/minipress

PTSD treatment
EMDR
SSRI
CBT

phases of EMDR
see purple book

Panic attack vs disorder
panic attack: acute/seldom

disorder: chronic leads to feelings of impending doom
propanolol

Tourette dx criteria
2 motor tic
1 vocal tic
for ONE year (even if ? says 6 months) its still most likely the correct answer

NT in tourette
DA-da dysfunction is major NT in tourette
NE
S

tics can be caused by what type of medication
stimulant

how do you treat a kid with tics for adhd
non stimulant

guanfacine and clonidine are good choice for adhd in kid with tics
TRUE

Meds for tourettes
FDA approved
haldol
abilify
pimozide
guanfacine
clonidine
esp if kid is having impulse control problems

are tics in kids normal
yes
normal.
normally by teen years they disappear. no treatment unless meeting dsm criteria for tourettes. tourettes needs trmt. 2 motor 1 vocal

brain region affected in adhd
frontal cortex

basal ganglia-

dorsolateral PFC–executive function, attention, cognitive processes (problem solving, working memory)

reticular activating

OCD NT’s
S
NE

Pandas
causes OCD
from strep infection
treat with SSRI prozac in kids, in adults use sertraline.
adults can also get TCA like clomipramine

DMDD (disruptive mood dysregulation disorder)
in kids, mood d/o 6-17yo
kid has bipolar disorder but is not 18 yet

moody for no reason
irritible for no reason
tantrums for no reason
Mad/Sad for no reason

IED (intermittent explosive disorder)
reacts grossly out of proportion to the situation

violent
aggressive
HAVE REMORSE, feel bad about it all later

irritibility, depression, labile mood…utilize a________
mood questionnaire

nightmares in children can be genetic or psychological
assess the family tree/hx for this

GAD (Generalized Anxiety Disorder)
AT LEAST SIX MONTHS
If less it is ASD

Autism Spectrum Disorder NTs
Gaba
glutamate
SE

impaired communication
poor cognition
broken mirror theory of autism-

what two things do you have to have to diagnose adhd
a teacher and a parent evaluation

Do stimulants increase DA?
Yes, so don’t give a vmat 2 and a stimulant together. wipes one another out

P450 Inducer meds
carbamazepine
rifampin
alcohol
phenytoin
griseofulvin
phenobarbital
sulfonylureas

P450 inhibitor
Valproate
ketoconazole
isoniazid
sulfonomides
choramphenical
amiodarone
erythromycin
quinidine
grapefruit juice

pneumonic for inducers
CRAP GPS induces my rage
Carbamazepine
Rifampin
Alcohol
Phenytoin
Griseofulvin
Phenobarbital
Sulfonylureas

450 subtypes
1A2
2E1
2C9
2D6
3A4
1A2. AcetAminophen- 1 big word with 2A
2E1 – 21 years to drink ETOH
2C9- Warfarin factors 2, C, 9
2D6- 2D echo-cardiac drug metabolism
3A4- most common, metabolizes up to 60% of ALL medications, so if its not one of the above it is likely 3A4.

What NT are involved with OCD
NE
SE

Trmt med for kid with ocd
SSRI: prozac

trmt med for adult with ocd
ssri: sertralin
or TCA (clomipramine)

GAD pneumonic
WATCHERS x 6 months
Worry
Anxiety
Tension
Concentration
Hyperarousal
Energy Loss
Restlessness
Sleep issues

Stages of change
precontemplation: Not acknowledged an issue. ” I don’t have a drug problem”

contemplation: acknowledges, but NOT YET ready to change. “i know heroin is killing me, but i have no time for rehab”… some self doubt

preparation: acknowledges issue, intends to eventually act. “Ive been thinking about rehab or buprenorphine, what do you think?” No action or behavioral modification yet… researching

action: behavioral change occurs. “I’m in inpatient rehab and I’m feeling much better!”

maintenance: effort made to sustain the change. “Ive been on buprenorphine for a year and I’m still doing well”

relapse: reversion to a previous stage… goes back to a prior stage to start over.

Autism hallmarks
impaired communication
impaired social interaction
poor eye contact
sensory issues
poor cognition

Broken mirror theory of autism-responsible for childs presentation of autism symptoms
mirror neuron is dysfunctional

Risk factors for autism
male
intellectual disability
genetic loading-inc risk with family member with autism

screeners for autism?
M-Chat: modified checklist for autism

ADOS-G: autism diagnostic observation schedule-generic

ASQ: ages and stages questionnaire

nightmares in children
can be genetic
so ASSESS family for same problem

NT in Autism Spectrum disorder
Gaba and Glutamate

If patient presents with irritibility, personality changes, check their:
Vit B12 and folic acid

if ? presents lack of coordination, slowing, motor symptoms, and apathy, depression, irritability think about this dx
subcortical dementia

cortical dementia effects mostly…
memory and language

cognitive deficits
motor, behavioral

pseudo dementia
cognitive screening
present with:
agitation
irritability

what disorder does a provider hear a lot of “I don’t know answers”
pseudo dementia

aphasia is associated with which region of the brain
prefrontal cortex

visual hallucinations are associated with which type of dementia
lewy body

frontotemoporal dementia
frontal lobe
picks disease
personality changes
behavioral
language (slurred)

Signs of lead abuse
developmental delay
learning difficulties
irritability
loss of appetite
weight loss
sluggishness
fatigue
abd pain
vomiting
constipation
hearing loss
seizures
pica

TEST LEAD LEVELS

Components of the COW Scale
Pulse
sweating
restlessness
pupil size
bone aches
running nose
tearing
GI upset
yawning
tremors
anxiety/irritability
goosebumps

anterior cingulate is responsible for what
cognitive functions
decision making
emotions
impulse control

NE is produced in the locus ceruleus and the …
medullary reticular formation

SE is produced in the_________? And is involved in __?
raphe nuclei of the brainstem/sleep

DA is produce in the substantia nigra and __?
the ventral tegmental area VTA
and nucleus accumbens

Acetylcholine is synthesized by the basal nucleus of _?
Meynert

NT in mood disorders
SE and NE

risk factors for osteoporosis
smoking
caffeine
lack of exercise
diet low in cal and vit D

What is the hippocampus responsible for?
ST to LT memory
emotions
stress
learning

Amygdala
fear
anger
anxiety
aggression
stress

cerebellum
balance

anterior cingulate
cognitive functions
decision making
empathy
impulse control
emotions
decision making

frontal lobe issues
social skills
tumor here can cause behavioral problems

If you are caring for an infant that is dying and the parents are present, what action should you take
give the infant to the parents to hold and
grieve their loss

osteoporosis can be prevented by a diet rich in what? and other ways to prevent
calcium and vitamin d
weight bearing exercises
not using tobacco

Cognitive theory basics
trying to replace automatic negative beliefs/irrational thoughts with positive / functional thoughts

Humanistic therapy basics
Person centered
self actualization
self directed growth

Behavioral therapy
problem solving techniques
role playing
skills training
relaxation

SKILLS SKILLS

interpersonal therapy (IPT)
interpersonal distress with lots of people and in several settings.
also used in marital conflict for a 12-16 weeks.
Think relationship distress with this

Family systems therapy
triangles
triangulation
self differentiation
genograms

structural family therapy
hierarchies keyword
structural mapping
genograms

strategic family therapy
problem focused/symptom focused
paradoxical strategies
or a straight forward directive
Reframe patients belief system

solution focused family therapy
miracle question
miracle solution
exception based finding questions
scaling questions

Appreciative inquiry
focused on the strengths of client

if question answers give closed ended questions and one open ended question and all are appropriate, choose open ended because open ended questions promote the relationship
promote the relationship between client/provider. if they won’t talk, you can f/u with closed ended questions if the child remains avoidant… stays avoidant, then go to parents as a last resort

therapy session with a couple and only one party shows up, what do you do?
reschedule both of them. they must both be present.

if someone has lost someone, ask them….?
how is that loss affecting you…how do you feel about it?
Not affected? maybe you don’t need to focus on it too much

If client is moving to another state, how can you assist in not abruptly stopping medications?
give plenty of meds until they can see a new provider…maybe three month supply

Sibling abuse 5yo sodomized by his 15yo borther reported to you. Inform parent to , and call?
tell parents to separate the siblings
cps

suspect rheumatoid arthritis, check…..?
ESR level, which indicates inflammation

PICOT
Population
Intervention
Comparison
Outcome
Time

Level 1 evidence
systematic reviews of random control trials (RCTs) -highest internal validity due to randomizations

Level 2 evidence
at least 1 RCT

how do you continue to give evidence based care?
by reading current up to date journals

Shrill cry in infant
intracranial pressure; inconsolable

when do you start aftercare plan?
when patient is admitted

masturbating is normal in what age range? Freud stage?
3 to six year old
phallic stage

normal for young boys to develop breast enlargement/swelling/tenderness. Usually goes away in….?
six months

sex drive is driven by what hormone?
testosterone. so check this if c/o dec libido.

Testosterone decreases as we…?
age

if male or female c/o dec sex drive, chec…?
testosterone

Alcohol dehydrogenase
etoh enzyme
women have less than men so get intoxicated faster compared to men

best way to normalize the grieving process in children?
supportive group therapy

do psychoeducation

do not give prescriptive advise

these reflexes go away at what age:
grasp/palmar
moro/startle
babinski
grasp/palmar 5-6 months
moro/startle 5-6 months
babinski 9-12 months

what class of medications are PDE5?
sexual dysfunction
viagra
short absorption, rapidly absorbed

know diff between AN and BN
AN BMI is very low, high chance of hospital admission
VS changes like bradycardia. if parents won’t admit call cps. bmi to admit are 12, 13, 14.

Screen all adhd presentations for substance use

high risk for sub use in depression population
use early screening and early intervention

recommend accupuncture for
depression and pain

inform adolescents all info is confidential except
harm to self or others

what is habeus corpus
legal, not medical concept, that protects patients from unlawful hospitalization

substance induced psychosis has a very high rate of ?
HI. even more than ASPD.

disseminated encephalomyelitis
nervous system is affected

asymmetrical body movements

do a neuro exam

patient has a mood disorder. started depakote and been stable. BMI 25 now 30. eating alot of sweets and juices. recommendations?
A. switch med
b. dec depakote
c. ask them to exercise

how can you help your client remember something by anchoring their memory. how do you do thta?
ask a specific question that will provide an anchor to a timeline, like a wedding, bday, new job, etc.

ask what they know about medications before rxing
assesses their knowledge of meds

polypharmacy can lead to adverse side effects
assess medication hx

iatrogenic
an iatrogenic illness is an illness that is caused by a medication or physician.

tardive dys: caused by a med rx’d by a doc

post online forms to give responses to influence
policy making
advocacy

autoimmune diseases can lead to increased….?
cytokine levels

Desmopressin (DDAVP)
for nighttime bed wetting
reduces urine production esp at noc

guanfacine and clonidine is disliked by some patients due to?
have low tolerability due to lowers BP

cannot look up patietns information on Social media
violates trust

treatment for nighttime wetting under 12yo?
nonpharm
setting alarms
bladder training

if addicts are still blamining others they are not ready for….?
discharge. If still blaming others they are not ready. Need to take responsibility for their actions.

non compliant with medication?
intervention based on why they are not compliant

assess data before changing anything

if patient comes to your with concerns make sure you
validate their concerns to promote communication

come to office or discuss by phone

documentation for off label use? do this….
document why.
and document support for off label use.
MSL or trials

risk factors for sleep apnea
excessive weight
obesity
diabetes
smoking
HTN
narrowed airways

what is macrocytic anemia
vit B12 deficiency

MMSE high score means?
Good! low score is severe cognitive deficit

if depression is “severe” assess for
SI

MMSE scale
25-30 is normal
21-24 mild
10-20 moderate
0-9 severe

SLUM scale
27-30 Normal
21-26 Mild
0-20 dementia

HAM D
0-7 Normal
8-13 mild
14-18 moderate
19-22 severe
23 plus very severe

PHQ 9
0-4 normal
5-9 mild
10-14 moderate
15-19 moderate to severe
20-27 severe

Beck
0-9 normal
10-18 mild
19-29 moderate
30-63 severe

HAM A
under 17 mild
18-24 moderate
25 plus severe

GAD
0-4 normal
5-9 mild
10-14 moderate
15-21 severe

COWs
0-4 none
5-12 mild
13-24 moderate
25-35 mod to severe
35 plus severe

CIWA
0-9 none
10-15 mild
16-20 moderate
21 plus severe

cows criteria
mod: treatment at 13 and above for scheduled. consider buprenorphine or Suboxone. methadone consider safety concerns and under supervision
pulse
sweating
restlessness
pupil size-dilated
bone aches
runny nose
tearing
gi upset
yawning
tremors
anxi/irritable
goosebumps

ciwa
scheduled medication starts at score of 15
Nausea
vomiting
tremor
sweats
anxiety
agitation
tactile disturbances
auditory disturbances
HA
orientation

etoh abuse treatment
disulfram no etoh for two weeks after stopping/none 12 hours before starting

aversion therapy

naltrexone

idealization
exaggeration of good qualities of the person or object lost, followed by acceptance for the loss. widow exaggeraes how amzing her not so amazing husban is

false positive screen for pcp
tramadol
dextromethorphan
alprazolam
clonazepam
carvedilol
diphenhydramine

rationalization
always to work late and tells boss “I’m not the only one”

intellectualization
new diagnosis so you go home and do tons of research

scope of practice
varies state to state
defines role and action
varies broadly

standard of practice
determined by american nurses association
judges care given
reasonably prudent is standard

confidentiality
the assurance that messages and information are available only to those who are authorized to view them

exceptions to confidentiality
when info outweighs risk (think murder trial)

intent to harm self or others

info given to atty in litigation

releasing records to insurance companies

answering court orders, subpoenas, summons

Meeting state requirements for mandatory reporting of dx of condition

informed consent in apt with dementia
if client can repeat the benefits and risks o the med/

autonomy is:
right to self determination
EXCEPT
rennie vs klein: right to refuse medication if NOT-found incompetent

Rights of patient re environment
least restrictive first

just culture
safety
continual learning
designing safe systems
managing behavioral choices

Recovery model
treatment approach doesn’t really focus on the dx/illness
looking beyond the dx and search out abilities dreams goals
self direction
finding meaning in life despite the dx
person centered
individualized
NON LINEAR not step by step but based on growth setbacks and learning experiences

quality improvement projects focus on…
improving systems
decrease cost
improve productivity

a retrospective chart review would be a type of….
quality improvement initiative

PDSA is a process
not a project
an effective strategy

reflective practice
linking theory to practice
providing feedback

industry vs inferiority
school age 6-12
becoming industrious and getting confidence about what they can do… fx is feeling bad and inferior

intimacy vs isolation
age 20-35
in a loving relationship
starting a family
fx is feeling alone and isolated

piaget age 2-7
magical thinkeers
preoperational
“if i think about a new dress I will get one”

piaget 11 and up
formal operations
logical and abstract
algebra etc

primary prevention example
screening and community education

secondary prevention
crisis intervention
hotlines
disaster response

tertiary prevention
rehab
active treatment

Leave a Comment

Scroll to Top